You are on page 1of 62

Total Marks : 200

TEST - 19 Mark Scored : 0

1 Consider the following about Foreign Currency Non-Resident (FCNR) deposits.


1. It is a type of fixed deposit account opened for depositing income earned overseas.
2. The account must be held in Indian currency.
3. These accounts are taxed at special rates not applicable to general deposits in India.
4. The funds held in these accounts can be remitted back overseas subject to certain terms.
Select the correct answer using the codes below.
A. 1, 2 and 3 only
B. 1 and 4 only
C. 2, 3 and 4 only
D. 1, 2 and 4 only
User Answer :
Correct Answer : B
Answer Justification :

Justification: Statement 1: An FCNR account is a term deposit account that can be maintained by NRIs and PIOs
in foreign currency. Thus, FCNRs are not savings accounts but fixed deposit accounts.

These accounts can be opened jointly with other NRIs.

Opening of FCNR (B) in the name of individuals/ entities of Bangladesh/ Pakistan nationality/ ownership requires
prior approval of Reserve Bank of India.

Statement 2: The account is held in the currency in which the deposits are made.

Statement 3: These accounts are exempt from Income Tax in India.

Statement 4: With the rupee continuing to remain volatile, the Foreign Currency Non Resident (FCNR) account
can be a good option for Non Resident Indians (NRIs) looking to invest in India without worrying about currency
risks.

Q Source: Improvisation: Page 10: Economic Survey 2017-18

2 The Union Ministry of Commerce and Industry has launched a pilot Revenue Insurance Scheme for Plantation Crops
(RISPC) for protecting the growers from the risks of
1. Pest attacks
2. Yield loss associated with adverse weather
3. Non-realization of genuine insurance claim by farmers from private companies
4. Income loss caused by fall in domestic and international prices
Select the correct answer using the codes below.
A. 2 only
B. 1, 2 and 4 only
C. 1 and 3 only
D. 1, 2, 3 and 4
User Answer :
Correct Answer : B
Answer Justification :

Justification: RISPC announced in 2016 is improved form of the Price Stabilization Fund (PSF) Scheme, 2003
which was closed 2013.

It was launched for protecting growers of plantation crops from twin risks of yield loss due to pest attacks,

(C) Insights Active Learning. | All rights reserved. www.insightsias.com 1


Total Marks : 200
TEST - 19 Mark Scored : 0

adverse weather parameters etc. and income loss caused by fall in domestic and international prices.

It shall be covering tea, coffee, rubber, cardamom and tobacco plantations and shall be implemented by the
commodity boards.

It will be implemented on a pilot basis for two years i.e. till 2018 in eight districts in West Bengal, Kerala,
Andhra Pradesh, Assam, Karnataka, Sikkim and Tamil Nadu. On the basis of performance of the scheme in
pilot project, it will be considered for extension to other districts.

Q Source:
http://timesofindia.indiatimes.com/business/india-business/govt-approves-revenue-insurance-scheme-for-plantatio
n-crops/articleshow/57063427.cms

3 Consider the following about the Deccani School of Painting. Assertion (A): It developed as an offshoot of Mughal
School of painting. Reason (R): Several Mughal painters migrated to the Deccan during the period of Aurangzeb and
sought patronage there.
In the context of the above, which of these is correct?
A. A is correct, and R is an appropriate explanation of A.
B. A is correct, but R is not an appropriate explanation of A.
C. A is incorrect, but R is correct.
D. Both A and R are incorrect.
User Answer :
Correct Answer : C
Answer Justification :

Justification: In the Deccan, painting continued to develop independently of the Mughal style in the beginning.
However, later in the 17th and 18th centuries it was increasingly influenced by the Mughal style. So, A is wrong.

Later it made a significant contribution to the development of the Mughal style in North India.

Early centres of painting in the Deccan, during the 16th and 17th centuries were Ahmednagar, Bijapur and
Golconda.

Painting in Hyderabad started with the foundation of the Asafjhi dynasty by Mir Qamruddin Khan Nizam-
ul-Mulk in 1724 A.D.

Influence of the Mughal style of painting on the already existing early styles of Deccani paintings,
introduced by several Mughal painters who migrated to the Deccan during the period of Aurangzeb and
sought patronage there, was responsible for the development of various styles of painting in the Deccan at
Hyderabad and other centres. So, R is correct.

Q Source: http://ccrtindia.gov.in/miniaturepainting.php

4 The Economic Survey notes that the nominal GDP growth is generally not of particular interest to policymakers. This
is because
(C) Insights Active Learning. | All rights reserved. www.insightsias.com 2
Total Marks : 200
TEST - 19 Mark Scored : 0

1. Inflation erodes the real value of nominal GDP growth.


2. Nominal GDP numbers do not capture services sector contribution.
Which of the above is/are correct?
A. 1 only
B. 2 only
C. Both 1 and 2
D. None
User Answer :
Correct Answer : A
Answer Justification :

Justification: Statement 1: Suppose an economy produces only cars.

If the price of car this year is Rs. 1000, and 2 cars were produced, the GDP this year is Rs. 2000.

Next year if prices increase to 3000 and still 2 cars are produced, GDP will be Rs. 6000 without an actual increase
in production, thanks to inflation of 200%.

So, when inflation is adjusted in nominal GDP, we get real GDP which shows how much production (without the
influence of prices) has actually grown.

Statement 2: GDP means an estimate of both goods and services, so 2 is wrong.

Q Source: Improvisation: Chapter 1: Economic Survey 2017-18

5 The first Law Commission was established, under the Chairmanship of Lord Macaulay which recommended
codification of the Penal Code and the Criminal Procedure Code, by the
A. Government of India Act 1935
B. Charter Act of 1833
C. Law Commission Act, 1967
D. Act of 1919
User Answer :
Correct Answer : B
Answer Justification :

Learning: Law Reform in the ancient period was ad hoc and not institutionalised.

However, since the third decade of the nineteenth century, Law Commissions were constituted by the Government
from time to time.

The first law commission was established in 1834 under the Charter Act of 1833.

Thereafter, the second, third and fourth Law Commissions were constituted in 1853, 1861 and 1879
respectively helped adapt English Laws to Indian conditions.

The Indian Code of Civil Procedure, the Indian Contract Act, the Indian Evidence Act, the Transfer of
Property Act etc. are products of the labour of the first four Law Commissions.

The First Law Commission of Independent India in 1955 with the then Attorney-General of India, Mr. M.

(C) Insights Active Learning. | All rights reserved. www.insightsias.com 3


Total Marks : 200
TEST - 19 Mark Scored : 0

C. Setalvad, as its Chairman.

Q Source:
http://indiatoday.intoday.in/story/triple-talaq-muslim-supreme-court-uniform-civil-code-aimplb/1/878477.html

6 Artificial Food fortification can help tackle which of the following?


1. Increase low yield of cereal crops on arid and semi-arid lands
2. Prevent micronutrient deficiencies
Which of the above is/are correct?
A. 1 only
B. 2 only
C. Both 1 and 2
D. None
User Answer :
Correct Answer : B
Answer Justification :

Justification: Statement 1: Artificial Food fortification happens after the food has been grown and processed. Bio-
fortification occurs on field as improved and fortified varieties are grown on field (genetic engineering).

Statement 2: It is the process of adding micronutrients i.e. essential trace elements and vitamins into the food.

It is an integrated approach to prevent micronutrient deficiencies and complements other approaches to


improve health and nutrition.

This enrichment process has proven as an effective strategy to meet the nutritional needs of a large number
of people across various sections of the society, including the poor and underprivileged, pregnant women
and young children.

Food fortification does not require changes in existing food habits and patterns nor individual compliance.
It does not alter characteristics of food and is socio-culturally acceptable.

Q Source: http://indianexpress.com/article/india/fssai-constitutes-panel-to-identify-nutritional-gaps-4525531/

7 The recently launched Cartosat-2 Series Satellite has potential applications in


1. Road network monitoring and creating land use maps
2. Tracking climate change by assessing deep ocean current patterns
3. Beaming high frequency electromagnetic telephone and radio signals
Select the correct answer using the codes below.
A. 1 and 2 only
B. 1 only
C. 2 and 3 only
D. 1 and 3 only
User Answer :
Correct Answer : B
Answer Justification :

(C) Insights Active Learning. | All rights reserved. www.insightsias.com 4


Total Marks : 200
TEST - 19 Mark Scored : 0

Justification: Statement 1: It will operate from a polar Sun Synchronous Orbit.

It is earth observation satellite that will provide remote sensing services. Images sent by it will be useful for
coastal land use and regulation, road network monitoring and creation of land use maps, among others.

Statement 2: Cartosat will restrict mapping to surface regions of earth. So, 2 is wrong.

Statement 3: This is the function of a communication satellite, not a remote sensing satellite. We have covered the
concept in earlier tests. So, 3 is wrong.

Q Source: http://www.isro.gov.in/Spacecraft/cartosat-2-series-satellite

8 The river Brahmaputra receives a number of tributaries at its north and south banks, in the catchment area in India.
Which of these are its North Bank tributaries?
A. Jiadhal, Pranahita, Noa Dehing and Subansiri
B. Manas, Krishnai, Manjira and Digaru
C. Champamati, Dhansiri and Siang
D. Musi, Koyna and Tungabhadra
User Answer :
Correct Answer : C
Answer Justification :

Justification: Work by elimination. All options other than C contain at least one tributary of rivers other than
Brahmaputra. For e.g. Option D shows tributaries of Krishna river.

Option A - Pranahita is that of Godavari river. Option B Manjira too is that of Godavari.

The only left out option is C.

Learning: You can find the entire list here


http://www.india-wris.nrsc.gov.in/wrpinfo/index.php?title=Brahmaputra#Climatic_Conditions

The principal tributaries of the river joining from right are the Lohit, the Dibang, the Subansiri, the Jiabharali, the
Dhansiri, the Manas, the Torsa, the Sankosh and the Teesta whereas the Burhidihing, the Desang, the Dikhow, the
Dhansiri and the Kopili joins it from left. The major part of basin is covered with forest

Q Source: Improvisation: Chapter 1: India Yearbook 2017

http://www.india-wris.nrsc.gov.in/wrpinfo/index.php?title=Brahmaputra

9 Consider the following about Van Allen belts.


1. It is a layer of energetic charged particles held around earth due to earth’s gravitational field.
2. Particles coming from Solar winds and cosmic rays contribute in the formation of these belts.
3. The belt lies between mesosphere and stratosphere.
Select the correct answer using the codes below.
A. 1 and 2 only
B. 3 only
C. 2 only
D. 1 and 3 only
User Answer :
Correct Answer : C

(C) Insights Active Learning. | All rights reserved. www.insightsias.com 5


Total Marks : 200
TEST - 19 Mark Scored : 0

Answer Justification :

Justification: Statement 1: The term Van Allen belts refers specifically to the radiation belts surrounding Earth;
however, similar radiation belts have been discovered around other planets.

A radiation belt is a layer of energetic charged particles that is held in place around a magnetized planet, such as
the Earth, by the planet's magnetic field. So, 1 is wrong.

The Earth has two such belts and sometimes others may be temporarily created.

Statement 2: Most of the particles that form the belts are thought to come from solar wind and other particles by
cosmic rays. The belts are located in the inner region of the Earth's magnetosphere.

By trapping the solar wind, the magnetic field deflects those energetic particles and protects the Earth's
atmosphere from destruction.

Statement 3: Earth's two main belts extend from an altitude of about 1,000 to 60,000 kilometre above the surface
in which region radiation levels vary.

Mesosphere lies upto the height of about 85 km from earth.

Q Source: Improvisation: Chapter 1: Goh Cheng Leong - Certificate Physical and Human Geography

10 Western Himalayan ecoregion is drier and the forest is more fragmented than its Eastern Himalayan broadleaf forests
counterpart because
1. Eastern Himalayas receives more moisture from the Bay of Bengal monsoon.
2. Eastern Himalayas is at a higher average elevation than Western Himalayas.
Which of the above is/are correct?
A. 1 only
B. 2 only
C. Both 1 and 2
D. None
User Answer :
Correct Answer : A
(C) Insights Active Learning. | All rights reserved. www.insightsias.com 6
Total Marks : 200
TEST - 19 Mark Scored : 0

Answer Justification :

Justification: Statement 1: Western region is drier as Monsoon winds become moisture-less on reaching the
western sides of the sub-continent.

Statement 2: Elevation is not the reason, since eastern Himalayas too has very high peaks upto nearly 7000 metres.

Moreover, at lower elevations, Western ecoregion grades into Himalayan subtropical pine forests. At higher
elevations, it grades into Western Himalayan subalpine conifer forests as well as Northwestern Himalayan alpine
shrub and meadows and Western Himalayan alpine shrub and meadows.

Q Source: Improvisation: Chapter 1: India Yearbook 2017

11 This new group of painters rejected the art of Raja Ravi Varma as imitative and westernized. Also known as 'Indian
style of painting' in its early days, it was associated with Indian nationalism and later came to be supported by British
arts administrators like E. B. Havell. It was?
A. Rajasthani School of painting
B. Bengal school of painting
C. Cross miniature paintings
D. Pahari school of painting
User Answer :
Correct Answer : B
Answer Justification :

Learning: They Bengal group of painters declared that Ravi Verma’s style was unsuitable for depicting the
nation’s ancient myths and legends.

They turned to the inspiration to medieval Indian traditions of the miniature paintings and ancient art of mural
paintings in Ajanta Caves. The paintings of Ajanta and Bagh, Mogul, Rajput and Pahari miniatures provided the
models.

The continuity of earlier traditions was sought to be maintained by borrowing from legends and classical literature
like the Ramayana, the Mahabharata, Gita, and Puranas, the writings of Kalidasa and Omar Khayyam.

This eventually led to the development of the modern Indian painting.

Q Source: http://www.ccrtindia.gov.in/modernindianpainting.php

12 Consider the following with reference to the Flag code of India.


1. When carried in a procession with other flags, the Indian flag must be on the left side of all.
2. When the flag is in a moving column, persons present will stand at attention or salute as the flag passes them.
Which of the above is/are correct?
A. 1 only
B. 2 only
C. Both 1 and 2
D. None
User Answer :
Correct Answer : B
Answer Justification :

Justification: Statement 1: The flag, when carried in a procession or parade or with another flag or flags, should

(C) Insights Active Learning. | All rights reserved. www.insightsias.com 7


Total Marks : 200
TEST - 19 Mark Scored : 0

be on the marching right or alone in the centre at the front. The flag may form a distinctive feature of the unveiling
of a statue, monument, or plaque, but should never be used as the covering for the object.

When the Indian flag is flown on Indian territory along with other national flags, the general rule is that the Indian
flag should be the starting point of all flags.

Statement 2: Moreover, a dignitary may take the salute without a head dress. The flag salutation should be
followed by the playing of the national anthem.

Learning: Khadi or hand-spun cloth is the only material allowed to be used for the flag, and flying a flag made of
any other material is punishable by law with imprisonment up to three years, besides a fine.

The Flag should never touch the ground or water. Also, the Flag should not hold any objects other than flower
petals before unfurling.

No lettering should be inscribed on the flag. The flag cannot be used in uniforms.

Q Source: Improvisation: Chapter 2: India Yearbook 2016

13 Consider the following about the demographics of India. Assertion (A): The population of India as recorded at each
decennial census from the first Census has grown steadily ever since. Reason (R): Death rates have reduced
substantially owing to better health facilities as well as sex ratio has improved since first Census.
In the context of the above, which of these is correct?
A. A is correct, and R is an appropriate explanation of A.
B. A is correct, but R is not an appropriate explanation of A.
C. A is correct, but R is incorrect.
D. Both A and R are incorrect.
User Answer :
Correct Answer : D
Answer Justification :

Justification: The population of India, which at the turn of the twentieth century was around 238.4 million,
increased to reach 1,210.9 million by 2011.

The population of India as recorded at each decennial census from 1901 has grown steadily except for a
decrease during 1911-21. So, A is wrong.

Sex ratio, defined as the number of females per thousand males is an important social indicator to measure
the extent of prevailing equality between males and females in a society at a given point of time.

The sex ratio in the country has always remained unfavourable to females. It was 972 at the beginning of
the twentieth century and thereafter showed continuous decline until 1941.

The sex ratio from 1901-2011 has registered a 10 point increase at census 2011 over 2001; however, child
sex ratio has declined to 919 per thousand male.

So, R is wrong.

(C) Insights Active Learning. | All rights reserved. www.insightsias.com 8


Total Marks : 200
TEST - 19 Mark Scored : 0

Q Source: Chapter 1: India Yearbook 2017

14 This state is the largest provider of coconut, natural rubber and black pepper to Indian markets. It is largely an
agrarian economy. The state is?
A. Andhra Pradesh
B. Kerala
C. Maharashtra
D. Odisha
User Answer :
Correct Answer : B
Answer Justification :

Learning: Majority of the population in Kerala are dependent directly or indirectly on agriculture for their
livelihood.

The main crops grown in the state are paddy, coconut, pepper, cashew, cassava, and plantation crops like rubber.

Cash crops, like coconuts, rubber, tea and coffee, pepper and cardamom, cashew, areca nut, nutmeg, ginger,
cinnamon, cloves and the like, give the agriculture of Kerala a distinct flavour.

It is coconuts which bring the people their principal source of earning in agriculture. Nearly 70% of Indian output
of coconuts is provided by Kerala.

Q Source: Improvisation: Chapter 1: India Yearbook 2017

15 Consider the following about Terahertz (THz) transmitter technology recently seen in news.
1. It is the ultraviolet bandwidth update to the Li-Fi Technology.
2. It will use quantum communication technology to deliver deviceless messages.
Which of the above is/are correct?
A. 1 only
B. 2 only
C. Both 1 and 2
D. None
User Answer :
Correct Answer : D
Answer Justification :

Justification: Terahertz wireless is high-data-rate minimum-latency communications.

Scientists from Japan have developed terahertz (THz) transmitter technology, a next generation system
which can transmit digital data over 10 times faster than 5G mobile networks.

It will pave transmitting digital data at a rate exceeding 100 gigabits per second (gbps) over a single channel
using the 300-gigahertz band.

Its possible applications include fast download from contents servers to mobile devices and ultrafast
wireless links between base stations.

(C) Insights Active Learning. | All rights reserved. www.insightsias.com 9


Total Marks : 200
TEST - 19 Mark Scored : 0

None of the statements above (1 and 2) are related to this technology.

Q Source:
http://www.livemint.com/Science/PoLzbzLqhczrKOWW7hqyPN/Data-speeds-10-times-faster-than-5G-achieved-
by-terahertz-tr.html

16 Consider the following with reference to the National Atlas and Thematic Mapping Organisation (NATMO).
1. It is housed in the Ministry of Earth Sciences.
2. It has recently launched a Braille Atlas for visually impaired.
3. It collaborates with central and state government organisations to meet their map requirements.
Select the correct answer using the codes below.
A. 1 and 2 only
B. 2 and 3 only
C. 1 only
D. 1, 2 and 3
User Answer :
Correct Answer : B
Answer Justification :

Justification: Statement 1: It is under the Department of S&T.

Statement 2: It is world’s first braille atlas for the visually impaired. In the Braille Atlas, maps are raised and
embossed with simple point and lines symbols to facilitate users to realize the location and area coverage easily in
terms of distinguishable shapes and textures. It is made using indigenously developed silk-screen painting
technology.

The Atlas will be distributed free of cost to various schools across the country for educating the students with
visual challenges.

Statement 3: NATMO is the premier agency of applied Geo-Spatial technology and pioneer in Thematic Mapping
and Atlas Cartography.

It is engaged in preparing different atlases, thematic maps and other documents on block level, district,
state, national and many other monograms.

Its thematic maps and data are used by the decision makers at districts and sub-divisions levels for
implementing the developmental schemes as well as by the researchers, students and common people.

Q Source: http://pib.nic.in/newsite/PrintRelease.aspx?relid=158369

17 Consider the following statements.


1. Linguistic minorities in India are determined on a state-wise basis.
2. Religious Minorities in India must be determined on national basis only.
Which of the above is/are correct?
A. 1 only
B. 2 only
C. Both 1 and 2
D. None
(C) Insights Active Learning. | All rights reserved. www.insightsias.com 10
Total Marks : 200
TEST - 19 Mark Scored : 0

User Answer :
Correct Answer : A
Answer Justification :

Justification: A linguistic minority is a group of people whose mother tongue is different from that of the majority
in the state or part of a state. Thus, the linguistic minorities are determined on a statewise basis. The same is
applicable for religious minorities as well.

In the 2002 judgment, in T.M.A. Pai Foundation & Others vs. the State of Karnataka and Ors, the Supreme
Court deliberated on the various contentions that the Centre, State, or a particular region within a State may
be considered as the basic unit for protection of the right of minorities.

The Court has set out the principle that minority status should be determined in relation to the population of
the State and not to India as a whole.

It ruled that as the reorganisation of the States in India had been effected on linguistic lines, for the purpose
of determining a minority, the unit would be the State and not the whole of India.

Thus, religious and linguistic minorities, who have been placed on a par in Article 30, have to be considered
in terms of the State concerned.

Q Source: Chapter on Special Officer for Linguistic Minorities: Indian Polity: M Laxmikanth

18 Consider the following statements. Assertion (A): The First Lok Sabha met on the eve of the first republic
day. Reason (R): India ceased to be a British dominion post enactment of constitution.
In the context of the above, which of these is correct?
A. A is correct, and R is an appropriate explanation of A.
B. A is correct, but R is not an appropriate explanation of A.
C. A is correct, but R is incorrect.
D. A is incorrect, but R is correct.
User Answer :
Correct Answer : D
Answer Justification :

Justification: First Lok Sabha met in 1952 Under GV Mavalankar. So, A is wrong.

The Indian Independence Act 1947 merely converted India into a British dominion. It ceased to be dominion only
after 26th January 1950.

Till 1952, the Constituent assembly itself functioned as the law making body.

It was only after the first general elections that Lok Sabha was constituted and the assembly was relived of its
responsibility of acting as a dual body.

Q Source: Chapter 3: India Yearbook 2016

19 Consider the following about Departmental standing committees.

(C) Insights Active Learning. | All rights reserved. www.insightsias.com 11


Total Marks : 200
TEST - 19 Mark Scored : 0

1. The main objective of the standing committees is to secure greater financial accountability of the Executive to the
Legislature.
2. Such committees consist of members from Lok Sabha alone.
3. A minister is not eligible to be nominated as a member of any of the standing committees.
4. The term of office of each standing committee is one year from the date of its constitution.
Select the correct answer using the codes below.
A. 1 only
B. 1, 3 and 4 only
C. 2 and 4 only
D. 1, 2 and 3 only
User Answer :
Correct Answer : B
Answer Justification :

Justification: Statement 1: They assist the Parliament in debating the budget more effectively as they examine
department’s budget more closely than the Parliament could do. The 24 standing committees cover under their
jurisdiction all the ministries / departments of the Central Government.

Statement 2: Each standing committee consists of 31 members (21 from Lok Sabha and 10 from Rajya Sabha).
The members of the Lok Sabha are nominated by the Speaker from amongst its own members, just as the members
of the Rajya Sabha are nominated by the Chairman from amongst its members.

Statement 3: A minister is not eligible to be nominated as a member of any of the standing committees. In case a
member, after his nomination to any of the standing committees, is appointed a minister, he then ceases to be a
member of the committee.

Statement 4: The term of office of each standing committee is one year from the date of its constitution. Out of the
24 standing committees, 8 work under the Rajya Sabha and 16 under the Lok Sabha.

Q Source: Improvisation: Chapter 3: India Yearbook 2016

20 Mahabalipuram monuments are a major architectural feat of India. On a visit to the location, you are NOT likely to
find which of the following?
A. Limestone caves
B. Rock cut temples
C. Monolithic structures
D. Structural temples
User Answer :
Correct Answer : A
Answer Justification :

Justification & Learning: Option B: Of the nine monolithic temples found in Mahabalipuram, the most important
are Five Rathas known after the famous five Pandava brothers of the Mahabharata fame. These monuments are
carved out a single rock.

Option C: Recent excavations to the north and south of the Shore Temple have revealed rock-cut figures
representing religious themes of period prior to the construction to the temple. Besides, a monolithic Bhuvaraha, a
reclining image of Vishnu, the base of Durga shrine with deer and a square socket possibly to accommodate
mahastambha have also been exposed.

Learning: Most of the monuments like rock-cut rathas, sculptured scenes on open rocks like Arjuna's penance, the
caves of Govardhanadhari and Mahishasuramardini, the Jala-Sayana Perumal temple (the sleeping Mahavishnu or

(C) Insights Active Learning. | All rights reserved. www.insightsias.com 12


Total Marks : 200
TEST - 19 Mark Scored : 0

Chakrin at the rear part of the Shore temple complex) are attributed to the period of Narasimhavarman-I Mamalla.

Q Source: http://ccrtindia.gov.in/templearchitecture.php

21 A first-of-its-kind South Asia Training and Technical Assistance Centre (SARTTAC) was recently opened in India
by International Monetary Fund (IMF) for
1. Socio-economic capacity building in South Asia
2. Addressing immediate liquidity crunch in foreign exchange
Which of the above is/are correct?
A. 1 only
B. 2 only
C. Both 1 and 2
D. None
User Answer :
Correct Answer : A
Answer Justification :

Justification: It will work to support local member countries of South Asia viz. India, Bangladesh, Bhutan,
Maldives, Nepal and Sri Lanka to build human and institutional capacity and implement policies for growth and
poverty reduction.

SARTTAC is financed mainly by its six member South Asia countries (mentioned above) with additional
support from Australia, South Korea, European Union and United Kingdom.

It will allow the IMF to meet more of the high demand for technical assistance and training from the region.
It is expected to become the focal point for the delivery of IMF capacity development services to South
Asia.

The opening of SARTTAC is part of the MoU (between IMF and India and other nations) and marks a
major milestone in the partnership between the IMF and its member countries in the region.

Q Source: http://pib.nic.in/newsite/mbErel.aspx?relid=158430

22 Consider the following departments under the Government of India.


1. Department of Ex-Serviceman Welfare: Ministry of Personnel and Public Grievances
2. Department of Investment and Public Asset Management: Ministry of Heavy Industries and Public Enterprises
3. Department of States: Ministry of Parliamentary affairs
4. Department of Bio-technology: Ministry of Science and Technology
Select the correct matches using the codes below.
A. 1 and 2 only
B. 3 and 4 only
C. 4 only
D. 1, 3 and 4 only
User Answer :
Correct Answer : C
Answer Justification :

Justification: Statement 1: . Department of Ex-Serviceman Welfare is under Ministry of Defence.


(C) Insights Active Learning. | All rights reserved. www.insightsias.com 13
Total Marks : 200
TEST - 19 Mark Scored : 0

Statement 2: Department of Investment and Public Asset Management is under Ministry of Finance as larger
implications of disinvestment for the economy, fiscal deficit, public asset management etc need to be taken into
account

Statement 3: Department of States is under Ministry of Home Affairs, which should also be clear from the fact that
Home Minister heads the zonal councils meant for centre-state coordination.

Q Source: Chapter 3: India Yearbook 2016

23 The ‘AEW&C NETRA’ recently seen in news is related to


A. Airborne surveillance capacity of Indian air force
B. Bio-technological improvement in eye donation capacity
C. Early warning system to avoid Railways Collision
D. Low beam systems in motor vehicles to prevent traffic blinding
User Answer :
Correct Answer : A
Answer Justification :

Learning: The Indian Airforce (IAG) has formally inducted the first indigenously built Airborne Early Warning
and Control System (AEW&C) dubbed as NETRA.

The AEW&C NETRA has been indigenously developed by the Defence Research and Development
Organisation (DRDO).

AEW&C are airborne radar systems mounted on a carrier jet for airborne surveillance system i.e. to detect
and track aircraft, missiles, ships and vehicles and provide command and control to direct friendly forces.

It consists of active electronically scanned radar, secondary surveillance radar, electronic and
communication counter measures, LOS (Line of Sight) and beyond-LOS data link, voice communication
system.

The radar is mounted on the aircraft.

With this India, joins group of other countries such as United States, Russia and Israel which have
developed the AEW&C system.

Q Source:
http://www.hindustantimes.com/india-news/iaf-inducts-netra-aew-c-the-first-indigenously-developed-surveillance-
plane/story-UxyMeY9oH4XvamNEd9JNTP.html

24 Which of these is the final authority to approve a merger of the State bank of India with its subsidiaries?
A. Securities and Exchange Board of India (SEBI)
B. Reserve Bank of India (RBI)
C. Union Cabinet
D. Parliament of India and President
User Answer :

(C) Insights Active Learning. | All rights reserved. www.insightsias.com 14


Total Marks : 200
TEST - 19 Mark Scored : 0

Correct Answer : D
Answer Justification :

Learning: Option B: The RBI provides Guidelines for merger /amalgamation of private sector banks. PSBs are
regulated by Parliamentary enactments.

Option A: SEBI does not regulate banks, but security markets in India.

Option C and D: The Union Cabinet has recently approved the merger of SBI and five of its associate/subsidiary
banks. However, this will have to be approved by the Parliament.

For this purpose, the Cabinet has also approved the introduction of a Bill in Parliament to repeal the State Bank of
India (Subsidiary Banks) Act, 1959 and the State Bank of Hyderabad Act, 1956.

Learning: This action will result in the creation of a stronger merged entity. It will minimize vulnerability faced
by subsidiary banks to any geographic concentration risks. It will improve operational efficiency and economies of
scale resulting into in improved risk management and unified treasury operations. Existing customers of associate
banks will benefit from SBI’s global network.

Q Source: Merger of SBI and Subsidiaries

http://www.livemint.com/Money/Df9T8GTflb9toVxjX9TCfL/Shares-of-SBI-associate-banks-surge-as-high-as-13-
after-ca.html

25 The Brasilia declaration signed by the foreign ministers of IBSA pressed for
A. Greater security for women at workplaces
B. Improved macro-economic policy coordination in South-South countries
C. Urgent reforms in the United Nations, especially the Security Council
D. Climate change mitigation funding support by developed countries
User Answer :
Correct Answer : C
Answer Justification :

Learning: It is one of the few important agreements signed by IBSA, hence the question.

In 2003, the Brasilia declaration was signed by the foreign ministers of India, Brazil and South Africa.

They agreed on the urgent need for reforms in the United Nations, especially the Security Council.

The declaration was of the view that the United Nations Security Council should reflect the current world scenario.

It also touched upon the subjects of international terrorism, trans-national crime and illegal arms dealing, stating
that such threats to international peace must effectively tackled with respect for the sovereignty of States and for
International Law

Learning: The IBSA Dialogue Forum aims to promote South-South cooperation and build consensus on issues of
international importance. It also aims at increasing the trade opportunities among the three countries, as well as
facilitate the trilateral exchange of information, technologies and skills

Q Source: Improvisation: Chapter 3: India Yearbook 2016

26 Which of these bodies find mention in the Constitution of India?

(C) Insights Active Learning. | All rights reserved. www.insightsias.com 15


Total Marks : 200
TEST - 19 Mark Scored : 0

1. District Planning Committee and Metropolitan Planning Committee


2. Inter-State Trade and Commerce Commission
3. National Backward Classes Commission
4. Official Language Commission and Official Language Committee of Parliament
Select the correct answer using the codes below.
A. 1 and 2 only
B. 3 and 4 only
C. 1, 2, 3 and 4
D. None of the above
User Answer :
Correct Answer : C
Answer Justification :

Justification: Statement 2: While there is a provision to establish the commission, it has not been established till
date. Even the Punchhi commission recommended the same.

Statement 4: As defined in the Article-344 of the Constitution, it shall be the duty of the Commission to make
recommendations to the President as to:

the progressive use of the Hindi language for the official purposes of the Union;

restrictions on the use of the English language for all or any of the official purposes of the Union;

the language to be used for all or any of the purposes mentioned in Article 348;

the form of numerals to be used for any one or more specified purposes of the Union;

Learning: These are some of the Articles Related to Constitutional Bodies (which are not very popular or widely
known)

165. Advocate-General of the State

243-I. State Finance Commission

243-K. State Election Commission

243ZD. District Planning Committee

243ZE. Metropolitan Planning Committee

307. Inter-State Trade and Commerce Commission

324. Election Commission

(C) Insights Active Learning. | All rights reserved. www.insightsias.com 16


Total Marks : 200
TEST - 19 Mark Scored : 0

339. Scheduled Areas and Scheduled Tribes Commission

340. Backward Classes Commission

344. Official Language Commission and Official Language Committee of Parliament

350B. Special Officer for Linguistic Minorities

Q Source: Chapter on NCST: Indian Polity: M Laxmikanth

27 Consider the following about Paramparagat Krishi Vikas Yojana (PKVY).


1. The scheme promotes organic farming in villages based on cluster approach.
2. The scheme is a component of the National Mission of Sustainable Agriculture (NMSA).
Which of the above is/are correct?
A. 1 only
B. 2 only
C. Both 1 and 2
D. None
User Answer :
Correct Answer : C
Answer Justification :

Justification: Statement 1: The Scheme envisages promotion of commercial organic production through certified
organic farming. The produce will be pesticide residue free and will contribute to improve the health of consumer;
raise farmer's income and motivate the farmers for natural resource mobilization for input production.

Groups of farmers would be motivated to take up organic farming under Paramparagat Krishi Vikas Yojana
(PKVY).

Fifty or more farmers will form a cluster having 50 acre land to take up the organic farming under the scheme.
Every farmer will be provided Rs. 20,000 per acre in three years for seed to harvesting of crops and to transport
produce to the market.

Statement 2: Paramparagat Krishi Vikas Yojana” is an elaborated component of Soil Health Management (SHM)
of major project National Mission of Sustainable Agriculture (NMSA).

There will be no liability on the farmers for expenditure on certification.

Organic farming will be promoted by using traditional resources and the organic products will be linked with the
market.

Q Source: Chapter 4: India Yearbook 2016

28 The Right to Information Act, 2005 empowers the citizens, promotes transparency and accountability in the working
of the Government. Consider the following about it.
1. The Act is applicable only to constitutional or statutory authorities.
2. Under the Act, a citizen can visit public office and take certified samples of material held by the public authority.
3. The Act requires public authorities to publish certain types of information publicly on a suo moto basis.

(C) Insights Active Learning. | All rights reserved. www.insightsias.com 17


Total Marks : 200
TEST - 19 Mark Scored : 0

Select the correct answer using the codes below.


A. 1 and 2 only
B. 2 and 3 only
C. 3 only
D. 1 only
User Answer :
Correct Answer : B
Answer Justification :

Justification: Statement 1: The Act gives all the citizens the right to seek information held by any authority or
body or institution of self government established by Constitution/law/notification etc by Central/state
government.

Bodies owned, controlled or substantially financed by the Central Government or a State Government and non-
Government organizations substantially financed by the Central Government or a State Government also fall
within the definition of public authority.

The financing of the body or the NGO by the Government may be direct or indirect. So, 1 is clearly incorrect.

Statement 2 and 3: The right includes inspection of work, documents and records; taking notes, extracts or
certified copies of documents or records, taking certified samples of material held by the public authority or held
under the control of the public authority.

It also includes information relating to any private body which can be accessed by the public authority under any
law for the time being in force. There are some categories of information which each public authority is required
to disclose pro-actively on suo moto basis.

Q Source: Chapter 3: India Yearbook 2016

29 Consider the following statements.


1. The largest volcano in our Solar System is found on Earth.
2. Ice cannot be found anywhere outside of earth in our solar system.
Which of the above is/are correct?
A. 1 only
B. 2 only
C. Both 1 and 2
D. None
User Answer :
Correct Answer : D
Answer Justification :

Justification: Statement 1: A shield volcano on Mars, Olympus Mons, is the largest volcano in the Solar System.

It is three times taller than Mount Everest and about five hundred km in diameter.

The surface of Venus is dominated by volcanic features and has more volcanoes than any other planet in the
Solar System.

Mercury is quiter as compared to these two planets, but volcanoes do exist on it.

(C) Insights Active Learning. | All rights reserved. www.insightsias.com 18


Total Marks : 200
TEST - 19 Mark Scored : 0

Statement 2: There is evidence for the existence of ice, as well as water, on the surface of planets other than earth.

In 2012, NASA’s Messenger confirmed the discovery of ice in permanently shadowed craters near Mercury
planet's North pole.

Astronomers have detected that the atmosphere of Venus as well as of Mars contains water vapour.

But, there isn’t any water on the surface of Venus, in form of rivers, lakes or oceans.

New findings from NASA's Mars Reconnaissance Orbiter (MRO) provide the strongest evidence yet that
liquid water flows intermittently on present-day Mars.

Q Source: Improvisation: Chapter 1: Goh Cheng Leong - Certificate Physical and Human Geography

30 Consider the following about the Pala School of Art.


1. It developed in the Magadha region of Southern Bihar in form of miniature paintings.
2. It started flourishing after the destruction of the Buddhist monasteries at the hands of Muslim invaders due to
increased patronage by Emperors.
Which of the above is/are correct?
A. 1 only
B. 2 only
C. Both 1 and 2
D. None
User Answer :
Correct Answer : A
Answer Justification :

Justification: The earliest examples of miniature painting in India exist in the form of illustrations to the religious
texts on Buddhism executed under the Palas of the eastern India and the Jain texts executed in western India
during the 11th-12th centuries A.D.

Statement 1: The Pala dynasty came to power around 750 A.D. The Pala school of art first flourished in the
Magadha region of Southern Bihar, the homeland of Buddhist religion. Not surprisingly, the majority of early
Pala-period remains are Buddhist

The Pala painting is characterised by sinuous line and subdued tones of colour. It is a naturalistic style which
resembles the ideal forms of contemporary bronze and stone sculpture, and reflects some feeling of the classical
art of Ajanta.

Statement 2: The Pala art came to a sudden end after the destruction of the Buddhist monasteries at the hands of
Muslim invaders in the first half of the 13th century. Some of the monks and artists escaped and fled to Nepal,
which helped in reinforcing the existing art traditions there

Q Source: http://ccrtindia.gov.in/miniaturepainting.php

31 The State Emblem of India is an adaptation from the Sarnath Lion Capital of Asoka. Consider the following about
the Sarnath figure.

(C) Insights Active Learning. | All rights reserved. www.insightsias.com 19


Total Marks : 200
TEST - 19 Mark Scored : 0

1. In the original figure, the Lion Capital has six lions mounted back to back on a circular abacus.
2. The abacus of the figure rests on a bell shaped lotus.
3. Apart from Lion, only horse and elephant are the other animals that have been depicted on the figure.
Select the correct answer using the codes below.
A. 1 and 2 only
B. 2 only
C. 3 only
D. 1, 2 and 3
User Answer :
Correct Answer : B
Answer Justification :

Justification: Statement 1: There are only four lions resting on the circular abacus in the original figure.

Statement 2 and 3: The frieze of the abacus is adorned with sculptures in high relief of an elephant, a galloping
horse, a bull and a lion separated by intervening Dharma Chakras. The abacus rests on a bell shaped lotus.

Learning: The profile of the Lion Capital showing three lions mounted on the abacus with a Dharma Chakra in
the centre, a bull on the right and a galloping horse on the left, and outlines of Dharma Chakras on the extreme
right and left was adopted as the State Emblem of India on January 26, 1950.

The bell-shaped lotus was omitted.

The motto “Satzjameva Jayate”-Truth alone triumphs-written in Devanagari script below the profile of the Lion
Capital is part of the State Emblem of India.

Q Source: Chapter 2: India Yearbook 2016

32 This national park is situated at the convergence point of Himalayan ranges, Zanskar and Western and Eastern
Himalayas. It is in the UNESCO World Network of Biosphere Reserves. The park is?
A. Jim Corbett
B. Dachigham
C. Valley of Flowers
D. Singalila Park
(C) Insights Active Learning. | All rights reserved. www.insightsias.com 20
Total Marks : 200
TEST - 19 Mark Scored : 0

User Answer :
Correct Answer : C
Answer Justification :

Learning: The Valley of Flowers has gained importance as a region containing a diversity of alpine flora,
representative of the Western Himalayan alpine shrub and meadows ecoregion.

The rich diversity of species reflects the valley's location within a transition zone between the Zanskar and
Great Himalayas ranges to the north and south, respectively, and between the Eastern Himalaya and
Western Himalaya flora

Nomination dossiers for this site have been sent to the World Heritage Centre for inscription on the World
Heritage list of UNESCO.

It is home to rare and endangered animals, including the Asiatic black bear, snow leopard, musk deer,
brown bear, red fox, and blue sheep.

With the Nanda Devi National Park to the east, it forms a part of nanda Devi biosphere reserve.

Q Source: Chapter 5: India Yearbook 2017

33 Ek Bharat Shreshtha Bharat is an initiative of


A. Cultural exchange and understanding between various states of India
(C) Insights Active Learning. | All rights reserved. www.insightsias.com 21
Total Marks : 200
TEST - 19 Mark Scored : 0

B. Lowering of inter-state trade barriers to boost economic integration


C. Increasing awareness of India’s rich heritage abroad in non-resident Indian citizens
D. Addressing all the calls for secessionism and extremism in the hinterland of the country
User Answer :
Correct Answer : A
Answer Justification :

Learning: It was announced by Prime Minister in 2015 on the occasion of the 140th birth anniversary of Sardar
Vallabhbhai Patel.

The broad objectives of the initiative are:

To celebrate the unity in diversity of our nation and to maintain and strengthen the fabric of traditionally
existing emotional bonds between the people of our country;

To promote the spirit of national integration through a deep and structured engagement between all states
and union territories through a year-long planned engagement between states;

To showcase the rich heritage and culture, customs and traditions of either state for enabling people to
understand and appreciate the diversity that is India, thus fostering a sense of common identity;

To establish long-term engagements; and

To create an environment which promote learning between states by sharing best practices and experiences.

Q Source: Chapter 5: India Yearbook 2016

34 Consider the following with reference to ensuring the financial accountability of the legislature to the executive.
1. The CAG submits audit reports to the President.
2. The President lays CAG reports before the Public Accounts Committee.
3. Public Accounts Committee examines CAG reports submitted to Parliament.
4. Public Accounts Committee submits the audit report of CAG to the President after due examination.
Which of the above is/are the correct procedure(s) in practice?
A. 1 only
B. 2 and 4 only
C. 1 and 3 only
D. 1, 3 and 4 only
User Answer :
Correct Answer : C
Answer Justification :

Justification: The CAG submits three audit reports to the President—audit report on appropriation accounts, audit
report on finance accounts, and audit report on public undertakings.

The President lays these reports before both the Houses of Parliament. After this, the Public Accounts Committee
examines them and reports its findings to the Parliament.

(C) Insights Active Learning. | All rights reserved. www.insightsias.com 22


Total Marks : 200
TEST - 19 Mark Scored : 0

The appropriation accounts compare the actual expenditure with the expenditure sanctioned by the Parliament
through the Appropriation Act, while the finance accounts show the annual receipts and disbursements of the
Union government.

Q Source: Chapter on CAG: Indian Polity: M Laxmikanth

35 Consider the following statements about the inscriptions found at Aihole, Karnataka.
1. They are written in a single language and script.
2. They mention poet Kalidasa.
3. They do not entertain any matter other than the praise of Kings.
Select the correct answer using the codes below.
A. 1 and 2 only
B. 3 only
C. 2 only
D. 1, 2 and 3
User Answer :
Correct Answer : C
Answer Justification :

Justification: Statement 1: The inscription written in Sanskrit and it is in Kannada script.

Many inscriptions found at Aihole, but the inscription which found at Meguti Temple popularly known as Aihole
inscription, which has the significance in the history of India, witnessed for the many historical events of
Chalukyas.

Statement 2 and 3: There is a mention about the defeat of Harshavardhana by Pulikeshi II. And there is a mention
about the victory of Chalukyas on Pallavas, also mentioned about the shifting of the capital from Aihole to Badami
by Pulikeshi. There is mention about the poet Kalidasa.

Q Source: http://ccrtindia.gov.in/templearchitecture.php

36 Consider the following about ‘Core Banking Solutions (CBS)’.


1. Customers of CBS branches can avail various banking facilities from any other CBS branch located anywhere in
the world.
2. CBS makes it mandatory for banks to lend to priority sectors based on Reserve Bank of India guidelines.
Which of the above is/are correct?
A. 1 only
B. 2 only
C. Both 1 and 2
D. None
User Answer :
Correct Answer : A
Answer Justification :

Justification: Core Banking Solution (CBS) is networking of branches, which enables Customers to operate their
accounts, and avail banking services from any branch of the Bank on CBS network, regardless of where he
maintains his account.

The customer is no more the customer of a Branch. He becomes the Bank’s Customer.

Thus CBS is a step towards enhancing customer convenience through Anywhere and Anytime Banking.

(C) Insights Active Learning. | All rights reserved. www.insightsias.com 23


Total Marks : 200
TEST - 19 Mark Scored : 0

All CBS branches are inter-connected with each other.

Q Source: Improvisation: Chapter 8: India Yearbook 2017

37 Consider the following about Cyber appellate tribunal, India.


1. It has been established under the Information Technology Act, 2000.
2. It is not bound by the Civil Procedure Code and guided by the principles of natural justice.
Which of the above is/are correct?
A. 1 only
B. 2 only
C. Both 1 and 2
D. None
User Answer :
Correct Answer : C
Answer Justification :

Justification: Statement 1: It is the first and the only Cyber Appellate Tribunal in the country which has been
established by the Central Government.

Statement 2: The Cyber Appellate Tribunal has powers to regulate its own procedure including the place at which
it has its sittings. Every proceeding before the Cyber Appellate Tribunal shall be deemed to be a judicial
proceeding.

It has the same powers as are vested in a civil court under the Code of Civil Procedure.

Learning: The selection of the Chairperson and Members of the Tribunal is made by the Central Government in
consultation with the Chief Justice of India.

The Presiding Officer of a Cyber Appellate shall hold office for a term of five years from the date on which he
enters upon his office or until he attains the age of sixtyfive years, whichever is earlier.

Q Source: Improvisation: Chapter 8: India Yearbook 2017

38 Consider the following with reference to Central Indian paintings.


1. They take inspiration from Indian epics and religious texts.
2. They developed as an anti-Sanskrit cult to promote culture of tribal regions.
Which of the above is/are correct?
A. 1 only
B. 2 only
C. Both 1 and 2
D. None
User Answer :
Correct Answer : A
Answer Justification :

Justification: Statement 1: Unlike Mughal painting which is primarily secular, the art of painting in Central India,
Rajasthani and the Pahari region etc. is deeply rooted in the Indian traditions, taking inspiration from Indian epics,
religious texts like the Puranas, love poems in Sanskrit and other Indian languages, Indian folk-lore and works on
musical themes.

Statement 2: The cults of Vaishnavism, Saivism and Sakti exercised tremendous influence on the pictorial art of

(C) Insights Active Learning. | All rights reserved. www.insightsias.com 24


Total Marks : 200
TEST - 19 Mark Scored : 0

these places. Among these the cult of Krishna was the most popular one which inspired the patrons and artists. It
did not develop as anti-Sanskrit movement as mentioned in the statement above.

Q Source: http://www.ccrtindia.gov.in/visualarts.php

39 Consider the following about the National Anthem and National Song.
1. The National Anthem of India was first sung at the Calcutta Session of the Indian National Congress.
2. The National Anthem was adopted at the first Republic Day by the constituent assembly.
3. The National Song was first sung at the 1930 Lahore Session of the Indian National Congress.
Select the correct answer using the codes below.
A. 1 only
B. 1 and 2 only
C. 2 and 3 only
D. 1 and 3 only
User Answer :
Correct Answer : A
Answer Justification :

Justification: Statement 1 and 2: The song Jana-gana-mana, composed originally in Bengali by Rabindra Nath
Tagore, was adopted in its Hindi version by the Constituent Assembly as the National Anthem of India on January
24, 1950.

It was first sung on December 27, 1911 at the Calcutta Session of the Indian National Congress. The complete
song consists of five stanzas. The first stanza contains the full version of the National Anthem.

Statement 3: The song Vande Mataram, composed in Sanskrit by Bankimchandra Chatteii, was a source of
inspiration to the people in their struggle for freedom. It has an equal status with Jana-gana-mana. The first
political occasion when it was sung was the 1896 session of the Indian National Congress.

Q Source: Chapter 2: India Yearbook 2016

40 Which of the following functions have been specified for the National Commission for STs with relation to the
protection and welfare of STs?
1. Measures to be taken to promote the traditional practice of shifting cultivation
2. Measures to be taken over conferring ownership rights in respect of minor forest produce to STs living in forest
areas
3. Measures to be taken to safeguard rights of the tribal communities over mineral resources
Select the correct answer using the codes below.
A. 1 only
B. 2 and 3 only
C. 1 and 3 only
D. None of the above
User Answer :
Correct Answer : B
Answer Justification :

Justification: In 2005, the President specified the following other functions of the Commission in relation to the
protection, welfare and development and advancement of the STs.

Statement 1: As per the notification, the commission should strive to reduce and eliminate shifting cultivation, not
encourage it, that lead to their continuous disempowerment and degradation of land and the environment.

(C) Insights Active Learning. | All rights reserved. www.insightsias.com 25


Total Marks : 200
TEST - 19 Mark Scored : 0

Learning: Others measures (apart from statements 2 and 3 are):

Development of tribals and to work for more viable livelihood strategies

Improve the efficacy of relief and rehabilitation measures for tribal groups displaced by development
projects

Prevent alienation of tribal people from land and to effectively rehabilitate such people in whose case
alienation has already taken place

Elicit maximum cooperation and involvement of tribal communities for protecting forests and undertaking
social afforestation

Ensure full implementation of the Provisions of Panchayats (Extension to the Scheduled Areas) Act, 1996

The Commission presents an annual report to the President. It can also submit a report as and when it thinks
necessary.

Q Source: Chapter on NCST: Indian Polity: M Laxmikanth

41 The State is used as a transit route for illegal movement of weapons, ammunition, narcotics, etc. from Myanmar to
Assam and Nagaland. Longding, Tirap and Changlang districts of the State are affected by the presence of Naga
insurgents groups. It is?
A. Meghalaya
B. Manipur
C. Arunachal Pradesh
D. Tripura
User Answer :
Correct Answer : C
Answer Justification :

Learning: Option A: Meghalaya is known for the violence that is largely confined to Garo hills perpetrated by the
Garo groups. Spillover of the activities of these groups was also visible in neighbouring districts of Goalpara and
Dhubri in Assam.

Option B: Majority of the violence in this state has been perpetrated by Valley based groups Politics of Blockades
and Bandhs continued to prevail and led to increasing fissures amongst the three major communities namely;
Meiteis-Nagas-Kukis. Demands of implementation of Inner Line Permit are central to the problems of this state.

Option C: The people of three eastern Arunachal districts - Tirap, Changlang and Longding - are living in fear due
to the presence of cadres of the two NSCN factions in the area, who resort to kidnapping, extortion and factional
feuds.

The problem of Arunachal is directly linked with that of Nagaland. The Centre should immediately resolve the
Naga issue to bring peace and normalcy to these districts.

Q Source: Chapter 9: India Yearbook 2017

(C) Insights Active Learning. | All rights reserved. www.insightsias.com 26


Total Marks : 200
TEST - 19 Mark Scored : 0

42 Consider the following statements.


1. It was built by the Chalukyas based on Dravidian style of architecture.
2. It is a monolithic rock-cut temple and a part of the Ellora caves.
The above refers to?
A. Brhideshvara Temple
B. Kailasa Temple
C. Venkateshawara Temple
D. Lingaraja Temple
User Answer :
Correct Answer : B
Answer Justification :

Learning: In the middle of the 8th century the Rashtrakutas wrested power from the Chalukyas. They created the
greatest wonder of medieval Indian art in their Kailasa temple at Ellora. Quarried out of a hill and solid rocks, it is
sculptured on a grand scale.

The beautiful architectural rock sculpture from Ellora shows the marriage of Siva and Parvati. Siva holding
the hand of the bashful Parvati occupies the centre of the composition.

To the right Brahma, the creator, is actively engaged in stirring up the flames of the sacred fire.

The parents of Parvati stand behind her to offer their daughter to the great god. A number of gods
assembled to witness the function are shown hovering above the principal figures. The dignified grace of
the divine couple and the gentle solemnity of the occasion have been portrayed by the sculptor with a
masterly skill.

Q Source: http://ccrtindia.gov.in/templearchitecture.php

43 ‘REER Index’ is known in the context of measuring


A. Corruption in the Central Government
B. Preparedness for digital revolution
C. Weighted average of Indian currency exchange rate
D. Innovation and technological evolution
User Answer :
Correct Answer : C
Answer Justification :

Learning: The real effective exchange rate (REER) is the weighted average of a country's currency relative to an
index or basket of other major currencies, adjusted for the effects of inflation.

The weights are determined by comparing the relative trade balance of a country's currency against each
country within the index.

This exchange rate is used to determine an individual country's currency value relative to the other major
currencies in the index, such as the U.S. dollar, Japanese yen and the euro.

(C) Insights Active Learning. | All rights reserved. www.insightsias.com 27


Total Marks : 200
TEST - 19 Mark Scored : 0

The REER takes into account any changes in relative prices and shows what can actually be purchased with
a currency.

Q Source: Chapter 1: Economic Survey 2017-18

44 Consider the following about the office of the Attorney General (AG).
1. Appointed by the president, he must be a person who is qualified to be appointed a judge of the Supreme Court.
2. The term of office of the AG is fixed by the Constitution giving him security of tenure.
3. His office is under the Ministry of Law and Justice and the AG reports the Union Law Minister.
Select the correct answer using the codes below.
A. 2 only
B. 1 and 3 only
C. 2 and 3 only
D. 1 only
User Answer :
Correct Answer : D
Answer Justification :

Justification: Statement 1: In other words, he must be a citizen of India and he must have been a judge of some
high court for five years or an advocate of some high court for ten years or an eminent jurist, in the opinion of the
president.

Statement 2: The term of office of the AG is not fixed by the Constitution. Further, the Constitution does not
contain the procedure and grounds for his removal. He holds office during the pleasure of the president. This
means that he may be removed by the president at any time.

Statement 3: During the prime ministership of Jawaharlal Nehru, a proposal was put forward by the Central
government that the office of the Attorney General be merged with the office of the law minister. It did not
materialise.

Learning: In the performance of his official duties, the Attorney General has the right of audience in all courts in
the territory of India.

Further, he has the right to speak and to take part in the proceedings of both the Houses of Parliament or their joint
sitting and any committee of the Parliament of which he may be named a member, but without a right to vote. He
enjoys all the privileges and immunities that are available to a member of Parliament.

Q Source: Chapter on AG: Indian Polity: M Laxmikanth

45 Consider the following about the Commonwealth Association for Public Administration and Management
(CAPAM).
1. CAPAM is a research partner of the United Nations General Assembly (UNGA) and UN Economic and Social
Council (UNECOSOC).
2. The Government of India is a member of CAPAM, which enables it to keep pace with the latest developments in
the field of public administration.
Which of the above is/are correct?
A. 1 only
B. 2 only
C. Both 1 and 2
D. None

(C) Insights Active Learning. | All rights reserved. www.insightsias.com 28


Total Marks : 200
TEST - 19 Mark Scored : 0

User Answer :
Correct Answer : B
Answer Justification :

Justification: Statement 1: With its headquarters at Ottawa, Canada, is an organization dedicated to strengthening
public management and consolidating democracy and good governance throughout the Commonwealth.

It was formed in 1994 as a result of decisions taken at the Commonwealth Heads of Government meeting.

Statement 2: The Ministry of Personnel, Public Grievances and Pensions, Government of India became an
institutional member of CAPAM in 1997.

The membership allows it participation in various programmes of CAPAM viz; International Innovations Awards
Programme, International Innovations Cascading Programme etc.

Q Source: Chapter 3: India Yearbook 2016

46 Consider the following statements. Assertion (A): Oceanic crust is less dense and thicker than continental crust.
Reason (R): Continental crust was formed from magma, unlike Oceanic crust which was largely a result of decay of
hydrocarbons.
In the context of the above, which of these is correct?
A. A is correct, and R is an appropriate explanation of A.
B. A is correct, but R is not an appropriate explanation of A.
C. A is correct, but R is incorrect.
D. Both A and R are incorrect.
User Answer :
Correct Answer : D
Answer Justification :

Justification & Learning: The crust overlies the solidified and uppermost layer of the mantle.

Oceanic crust is the result of erupted mantle material originating from below the plate, cooled and in most
instances, modified chemically by seawater.

It is primarily composed of mafic rocks, or sima, which is rich in iron and magnesium.

It is thinner than continental crust, or sial, generally less than ten km thick; however Oceanic crust is denser. So,
both A and R are incorrect.

Q Source: Chapter 2: Goh Cheng Leong - Certificate Physical and Human Geography

47 The chairman and members of National Human Rights Commission (NHRC) are appointed by the president on the
recommendations of a six-member committee. The committee does NOT include
A. Deputy Chairman of the Rajya Sabha
B. Union Home Minister
C. Leaders of the Opposition in both the Houses of Parliament
D. Chief Justice of India
User Answer :
Correct Answer : D
Answer Justification :

Learning: The six-member committee consists of the prime minister as its head, the Speaker of the Lok Sabha,
(C) Insights Active Learning. | All rights reserved. www.insightsias.com 29
Total Marks : 200
TEST - 19 Mark Scored : 0

the Deputy Chairman of the Rajya Sabha, leaders of the Opposition in both the Houses of Parliament and the
Central home minister.

Further, a sitting judge of the Supreme Court or a sitting chief justice of a high court can be appointed only after
consultation with the chief justice of India. The committee does not include the CJI. So, D is the answer.

The chairman and members hold office for a term of five years or until they attain the age of 70 years, whichever
is earlier. After their tenure, the chairman and members are not eligible for further employment under the Central
or a state government

Q Source: Chapter on NHRC: Indian Polity: M Laxmikanth

48 Consider the following statements.


1. Buddhist themes have been wall painted in Ajanta.
2. The rock shelters and caves of Bhimbetka host paintings of geometric patterns.
3. Bagh caves are renowned for mural paintings.
Select the correct answer using the codes below.
A. 1 only
B. 1 and 3 only
C. 2 and 3 only
D. 1, 2 and 3
User Answer :
Correct Answer : D
Answer Justification :

Justification: Statement 1: The rock shelters and caves of Bhimbetka have a large number of paintings. The oldest
paintings are considered to be 30,000 years old, but some of the geometric figures date to as recently as the
medieval period. The colours used are vegetable colors which have endured through time

Statement 2: Padmapani, Bodhisattva are some of the major examples. Some of them have been asked in previous
tests.

Statement 3: The paintings from Bagh caves in Madhya Pradesh correspond to some paintings of Ajanta.

However, these paintings are materialistic rather than spiritualistic.

Q Source: http://www.ccrtindia.gov.in/visualarts.php

49 Trade and cultural contacts existed between these cities and those of Mesopotamia. This conclusion was deduced
based on which of these evidence(s)?
1. Same female deity was worshipped in both these regions.
2. Similar seals and pottery occur at both these regions.
Which of the above is/are correct?
A. 1 only
B. 2 only
C. Both 1 and 2
D. None
User Answer :
Correct Answer : B
Answer Justification :

(C) Insights Active Learning. | All rights reserved. www.insightsias.com 30


Total Marks : 200
TEST - 19 Mark Scored : 0

Justification: Statement 1: People worshipped the Mother Goddess or Goddess of fertility. However, the deities
were not the same. This does not give any evidence for the existence of cultural or trade links between
Mesopotamia and Harappa.

Statement 2: The evidence is the occurrence of the seals, as well as similar carnelian beads, knobbed pottery, etc.,
at both places. Clay was the earliest medium in which man began to mould and we have discovered a large number
of terracotta figurines from these Indus Valley sites as well as the same at Mesopotamia sites.

Q Source: http://ccrtindia.gov.in/induscivilization.php

50 It is an autonomous body under the Ministry of Environment and Forests. It had launched a Corporate Social
Responsibility (CSR) initiative for allowing PSUs and Companies to adopt endangered species in India some time
back. It refers to?
A. Wildlife Institute of India (WII)
B. World Wild Life Fund (WWF), India
C. ENVIS Centre on Wildlife & Protected Areas
D. Forest Research Institute, India
User Answer :
Correct Answer : A
Answer Justification :

Learning: WII carries out wildlife research in areas of study like Biodiversity, Endangered Species, Wildlife
Policy, Wildlife Management, Wildlife Forensics, Spatial Modeling, Eco-development, and Climate Change.

It launched an initiative for allowing PSU and Companies to adopt endangered species such as Great Indian
bustard, Gharial, Lesser Florican, Snow Leopard etc.

Spending will be counted under CSR in Companies Act 2013. This is because Budget 2015 had reduced its
funding by 25%.

Q Source:
http://indianexpress.com/article/india/india-others/budget-cuts-make-govt-turn-to-psus-save-snow-leopard-gharial/

51 India’s first floating elementary school was recently inaugurated in which of the following states?
A. Odisha
B. Gujarat
C. Manipur
D. Mizoram
User Answer :
Correct Answer : C
Answer Justification :

Learning: The school named Loktak Elementary Floating School was inaugurated at Langolsabi Leikai of
Champu Khangpok floating village on Loktak Lake in Manipur.

The first of its kind school aims to provide education to drop outs students as well as illiterate adults.

Currently it will provide education to around 40 children from Class I-III by engaging two local teachers.

(C) Insights Active Learning. | All rights reserved. www.insightsias.com 31


Total Marks : 200
TEST - 19 Mark Scored : 0

It was opened under the initiative undertaken by All Loktak Lake Fisherman’s Union with the support of an
NGO People Resources Development Association (PRDA).

The school was established to arrest the sudden high dropout rate of children living in the Champu
Khangpok floating villages who were rendered homeless due to the recent evacuation of phumdis.

More than 700 floating huts were removed by Loktak Development Authority as part of clearing
encroachment in the lake under the Loktak Lake (Protection) Act 2006.

It had resulted in displacement of the fishing community and their children who were forced to discontinue
their education.

Q Source:
http://indianexpress.com/article/education/indias-first-floating-elementary-school-inaugurated-on-manipurs-loktak
-lake-4522383/

52 The cooler crust aside, the temperature on earth tends to increase as we dig inwards. The retained heat on earth can
be attributed to which of the following phenomena?
1. Frictional heating of earth’s material facilitated by gravitation
2. Heat from the decay of radioactive elements
Which of the above is/are correct?
A. 1 only
B. 2 only
C. Both 1 and 2
D. None
User Answer :
Correct Answer : C
Answer Justification :

Justification: Statement 1: Frictional heating is caused by denser core material sinking to the center of the planet
caused due to gravitation.

As material slides against each other, heat generated is retained deep within the earth and temperature remains
high.

Statement 2: It is estimated that about 50% of the heat given off by the Earth is generated by the radioactive decay
of elements such as uranium and thorium, and their decay products.

Further discussion becomes technical and is outside the purview of our syllabus.

Q Source: Improvisation: Chapter 1: Goh Cheng Leong - Certificate Physical and Human Geography

53 Consider the following statements about the Comptroller and Auditor General of India (CAG). Assertion (A): No
minister can represent the CAG in Parliament or be called upon to take any responsibility for any actions done by
him. Reason (R): The CAG is an autonomous body and not appointed by the President or Council of Ministers.
In the context of the above, which of these is correct?

(C) Insights Active Learning. | All rights reserved. www.insightsias.com 32


Total Marks : 200
TEST - 19 Mark Scored : 0

A. A is correct, and R is an appropriate explanation of A.


B. A is correct, but R is not an appropriate explanation of A.
C. A is correct, but R is incorrect.
D. Both A and R are incorrect.
User Answer :
Correct Answer : C
Answer Justification :

Justification: The CAG is appointed by the president of India by a warrant under his hand and seal. So, R is
wrong.

Moreover, the CAG, before taking over his office, makes and subscribes before the president an oath or
affirmation.

Ministers don’t represent CAG because CAG audits the expenditures sanctioned by the Ministers. If they represent
CAG, this will lead to conflict of interest. Further, CAG is a part of the Union Executive. Thus, A is correct and
reasonable.

Q Source: Chapter on CAG: Indian Polity: M Laxmikanth

54 International Covenant on Civil and Political Rights (ICCPR) does NOT cover which of these rights?
A. Freedom of speech
B. Right to livelihood and employment
C. Freedom of religion
D. Right to a fair trial
User Answer :
Correct Answer : B
Answer Justification :

Learning: It is a multilateral treaty adopted by the UNGA in 1966.

It commits its parties to respect the civil and political rights of individuals, including the right to life,
freedom of religion, freedom of speech, freedom of assembly, electoral rights and rights to due process and
a fair trial.

The ICCPR is part of the International Bill of Human Rights, along with the International Covenant on
Economic, Social and Cultural Rights (ICESCR) and the Universal Declaration of Human Rights (UDHR).

The ICCPR is monitored by the United Nations Human Rights Committee (a separate body to the United
Nations Human Rights Council), which reviews regular reports of States parties on how the rights are being
implemented.

States must report initially one year after acceding to the Covenant and then whenever the Committee
requests (usually every four years).

India is a party to the convention.

(C) Insights Active Learning. | All rights reserved. www.insightsias.com 33


Total Marks : 200
TEST - 19 Mark Scored : 0

Q Source: NOTES AND REFERENCES: Chapter on NHRC

55 Global Initiative for Academic Network (GIAN) aims at


1. Tapping the talent pool of scientists and entrepreneurs internationally to encourage their engagement with higher
education institutes in India
2. Developing an institutionalized framework at stopping out migration of students for higher studies and
employment abroad
Which of the above is/are correct?
A. 1 only
B. 2 only
C. Both 1 and 2
D. None
User Answer :
Correct Answer : A
Answer Justification :

Justification: Statement 1: GIAN seeks to encourage engagement (as mentioned in statement 1) with the institutes
of higher education in India so as to augment the country’s existing academic resources, accelerate the pace of
quality reform, and elevate India’s scientific and technological capacity to global excellence.

Statement 2: Some of the other objectives are:

To increase the footfalls of reputed international faculty in the Indian academic institutes.

Provide opportunity to our faculty to learn and share knowledge and teaching skills in cutting edge areas.

To provide opportunity to our students to seek knowledge and experience from reputed International
faculty.

To create avenue for possible collaborative research with the international faculty

To increase participation and presence of international students in the academic Institutes.

It is proposed to initiate the program under collaboration with various countries.

Q Source: Chapter 10: India yearbook 2017 and past current affairs -
http://indiatoday.intoday.in/education/story/gian-launched/1/535735.html

56 Consider the following statements. Assertion (A): Shallow―focus earthquakes may occur along transform
boundaries. Reason (R): Significant energy may be released where crustal plates slide past one another.
In the context of the above, which of these is correct?
A. A is correct, and R is an appropriate explanation of A.
B. A is correct, but R is not an appropriate explanation of A.
C. A is correct, but R is incorrect.
D. Both A and R are incorrect.
User Answer :

(C) Insights Active Learning. | All rights reserved. www.insightsias.com 34


Total Marks : 200
TEST - 19 Mark Scored : 0

Correct Answer : A
Answer Justification :

Justification: There are three types of plate boundaries: spreading zones, transform faults, and subduction zones.

At spreading zones, molten rock rises, pushing two plates apart and adding new material at their edges.

Divergent boundaries are those at which crustal plates move away from each other, such as at mid-oceanic
ridge.

Earthquakes are located along the normal faults that form the sides of the rift or beneath the floor of the rift.

However, transform faults are found where plates slide past one another. Shallow―focus earthquakes
occur along transform boundaries where two plates move past each other.

Q Source: Chapter 3: Goh Cheng Leong - Certificate Physical and Human Geography

57 ‘Unified license Policy’ is often heard in the context of


A. Locomotive training and certification
B. Spectrum allocation and use
C. Operation of private aircrafts in public airports
D. Allotment of coal blocks on First Come First Serve Basis (FCFS)
User Answer :
Correct Answer : B
Answer Justification :

Learning: One of the objectives of the National Telecom policy-2012 is "Strive to create One Nation - One
License" across services and service areas.

The Department of Telecom (DoT) has published guidelines for Unified Licenses for telecom companies in
2015.

As per the norms, licenses for telecom service providers will be delinked from spectrum.

Now telcom companies can offer all services that existing licenses permit while being able to share
spectrum and telecom infrastructure, something the earlier licensing regime did not permit.

This would help in the growth of telecommunications sector in India.

Q Source: Chapter 8: India Yearbook 2017

58 Consider the following statements.


1. Places in the east see the sun earlier than those in the west as the earth rotates from west to east.
(C) Insights Active Learning. | All rights reserved. www.insightsias.com 35
Total Marks : 200
TEST - 19 Mark Scored : 0

2. Sun is not directly overhead at all sunlit points on earth at the same time due to the curvature of the earth.
Which of the above is/are correct?
A. 1 only
B. 2 only
C. Both 1 and 2
D. None
User Answer :
Correct Answer : C
Answer Justification :

Justification: Statement 1 is self-explanatory.

Statement 2: If the earth was like a horizontal disc (where population lived only on the top surface), the whole
world would have sunrise and sunset at the same time.

Since it has a curvature, sun’s rays are slanted at most locations.

However, due to earth’s axial tilt Sun is overhead at least one point between the tropics in an year. In summer
months (India), it is tropic of cancer or regions near it. In winter months (India), it is the tropic of Capricorn or
regions near it.

Q Source: Chapter 1: Goh Cheng Leong - Certificate Physical and Human Geography

59 ‘Social audit’ is increasingly being seen as an important tool to improve accountability in governance. Which of
these can be assigned the responsibility for the social audit of a government scheme?
A. Comptroller General of India only
B. Gram Sabha or Gram Panchayat only
C. A community group or professional institution
D. Independent Evaluation Office (IEO) only
User Answer :
Correct Answer : C
Answer Justification :

Learning: Social audit means the process in which people collectively monitor and evaluate the planning and
implementation of a programme or scheme.

For e.g. a social audit was conducted by Society for Social Audit Accountability and Transparency
(SSAAT) in two districts viz., Khammam and Chittoor of Andhra Pradesh during 2012-13.

Encouraged by the outcome MHRD has issued detailed guidelines for conducting of social audit under Mid
Day Meal Scheme.

The states/UTs were advised to select the eminent institute available in their respective states for this
purpose.

Similarly, the Gram Sabha is entitled for the social audit of fund usage under MGNREGA as a mandatory
provision.

(C) Insights Active Learning. | All rights reserved. www.insightsias.com 36


Total Marks : 200
TEST - 19 Mark Scored : 0

Therefore, social audit is comprehensive and involves stakeholders other than government.

Q Source: Chapter 10: India Yearbook 2017

60 The organizations NSCN and ULFA, often seen in news, are known with respect to
A. Promotion of grass roots innovation societies in rural areas
B. Establishment of safe Medical and hygiene practices
C. Voluntary Border patrol guards attached to the Indian security agencies
D. Secessionist movements in North-east India
User Answer :
Correct Answer : D
Answer Justification :

Learning: The Nationalist Socialist Council of Nagaland/Nagalim (NSCN) is an extremist, Naga nationalist
insurgent group operating mainly in Northeast India, with minor activities in northwest Myanmar (Burma) until
2012.

The main goal of the organisation is to establish a sovereign Naga state, "Nagalim", which would consist of
all the areas inhabited by the Naga people in Northeast India and Northwest Myanmar.

The NSCN has been designated a terrorist organisation in India under the Unlawful Activities (Prevention)
Act, 1967.

The United Liberation Front of Assam is a separatist outfit operating in Assam, North East India. It seeks to
establish a sovereign Assam with an armed struggle in the Assam conflict. The government of India banned
the organisation in 1990 citing it as a terrorist organisation.

Q Source: Chapter 9: India Yearbook 2017

61 While recommending price policy of various commodities under its mandate, the Commission for Agricultural Costs
and Prices (CACP) keeps in mind
1. Cost of production
2. Domestic and international price trends
3. Inter-crop price parity
4. Terms of trade between agricultural and non-agricultural sectors
Select the correct answer using the codes below.
A. 2 and 4 only
B. 1 and 3 only
C. 1, 2 and 3 only
D. 1, 2, 3 and 4
User Answer :
Correct Answer : D
Answer Justification :

Justification: While recommending price policy of various commodities under its mandate, the Commission
keeps in mind demand and supply, cost of production, price trends in the market, both domestic and international,
inter-crop price parity (i.e. if one crop is too steeply priced as against the other), terms of trade (price ratios)
(C) Insights Active Learning. | All rights reserved. www.insightsias.com 37
Total Marks : 200
TEST - 19 Mark Scored : 0

between agricultural and non-agricultural sectors, likely implications of MSP on consumers of that product,
besides ensuring optimal utilization of natural resources like land and water.

It may be noted that cost of production is an important factor that goes as an input in determination of MSP but it
is not the only factor. Thus, recommending MSPs of various crops is not a ‘cost plus’ pricing exercise, though cost
is an important determinant.

Learning: CACP was set up with a view to evolving a balanced and integrated price structure, is mandated to
advise on the price policy (MSP) of 23 crops.

These include seven cereal crops (paddy, wheat, jowar, bajra, maize, ragi and barley), five pulse crops (gram, tur,
moong, urad and lentil), seven oilseeds (groundnut, sunflower seed, soyabean, rapeseed—mustard, safflower,
niger seed and sesamum), copra (dried coconut), coon, raw jute and sugarcane fair and remunerative prices (FRP)
instead of MSPA.

While recommending MSPs/FRP, CACP is called upon to ensure that the production patterns are broadly in line
with the overall needs (demand) of the economy.

Before preparing the pricing policy reports, the Commission seeks views of various state governments, concerned
national organizations and ministries.

Q Source: Chapter 4: India Yearbook 2016

62 The Actual Ground Position Line (AGPL) is the line that divides current positions of
A. Indian and Chinese troops in the North-east Frontier region
B. Chinese and Pakistani troops in the Xinjiang region
C. Indian and Pakistani troops in the Siachen Glacier region
D. Indian and Pakistani troops in the Kargil region
User Answer :
Correct Answer : C
Answer Justification :

Justification: The line extends from the northernmost point of the LOC (Line of Control) to Indira Col.

The AGPL is approximately 110 kilometres long.

This line runs across the edge of the Saltoro Ridge, which is a mountainous plateau.

The Indian soldiers hold on to the heights on the ridge, preventing the Pakistani soldiers from climbing up
to the Saltoro Range heights.

Learning: The Line of Actual Control (LAC) is a demarcation line that separates Indian-controlled territory from
Chinese-controlled territory.

The term Line of Control (LOC) refers to the military control line between the Indian and Pakistani controlled
parts Of J&K.

Q Source: Chapter 9: India Yearbook 2017

(C) Insights Active Learning. | All rights reserved. www.insightsias.com 38


Total Marks : 200
TEST - 19 Mark Scored : 0

63 The National Calendar based on the Saka Era is used along with the Gregorian calendar for which of the following
official communications?
1. Gazette of India
2. News broadcast by All India Radio
3. Government communications addressed to the public
Select the correct answer using the codes below.
A. 1 and 2 only
B. 2 only
C. 1 and 3 only
D. 1, 2 and 3
User Answer :
Correct Answer : D
Answer Justification :

Justification: The National Calendar, with Chaitra as its first month and a normal year of 365 days was adopted
from March 22, 1957 along with the Gregorian calendar for the following official purposes: (i) Gazette of India,
(ii) news broadcast by All India Radio, (iii) calendars issued by the Government of India and (iv) Government
communications addressed to the public.

Dates of the National Calendar have a permanent correspondence with dates of the Gregorian calendar, with 1
Chaitra falling on March 22 normally and on March 21 in Leap year.

Q Source: Chapter 2: India Yearbook 2016

64 The iron catastrophe was a postulated major event early in the history of Earth. It refers to
A. Erosion of minerals from earth’s crust due to solar winds
B. Sinking of iron to the centre of the earth leading to an overall restructuring
C. Large scale homogenization of concentrated iron deposits due to repeated ice ages
D. All of the above in chronological sequence
User Answer :
Correct Answer : B
Answer Justification :

Learning: The original accretion of the Earth's material into a spherical mass is thought to have resulted in a
relatively uniform composition.

While residual heat from the collision of the material that formed the Earth was significant, heating from
radioactive materials in this mass gradually increased the temperature until a critical condition was reached.

As material became molten enough to allow movement, the denser iron and nickel, evenly distributed
throughout the mass, began to migrate to the centre of the planet to form the core.

This happened about 4.5 billion years ago. 3) In the third step the compression itself began to heat the
interior of the Earth; also there was heat

Proceeding slowly at first, it sped up to catastrophic proportions - hence it is called the iron catastrophe.

(C) Insights Active Learning. | All rights reserved. www.insightsias.com 39


Total Marks : 200
TEST - 19 Mark Scored : 0

It was the iron catastrophe that set up the overall structure of the Earth.

Q Source: Chapter 2: Goh Cheng Leong - Certificate Physical and Human Geography

65 The National Institutional Ranking Framework (NIRF) as approved by the Ministry of Human Resource
Development (MHRD) includes which of these parameters for ranking?
1. Public Perception
2. Inclusivity
3. Graduation Outcomes
4. Research Output
Select the correct answer using the codes below.
A. 2 and 3 only
B. 2, 3 and 4 only
C. 1, 3 and 4 only
D. 1, 2, 3 and 4
User Answer :
Correct Answer : D
Answer Justification :

Justification: This framework outlines a methodology to rank institutions across the country. The methodology
draws from the overall recommendations broad understanding arrived at by a Core Committee set up by MHRD,
to identify the broad parameters for ranking various universities and institutions.

The parameters broadly cover “Teaching, Learning and Resources,” “Research and Professional Practices,”
“Graduation Outcomes,” “Outreach and Inclusivity,” and “Perception”.

Learning: For some of the parameters (like Research, Patents etc.) the data will be populated from internationally
available Data Bases (like Scopus, Web of Science, the Indian Science Index or other suitable sources as deemed
appropriate by NIRF).

Open Universities and Affiliating Universities (whether State or Centre approved/funded) will not normally be
registered for ranking.

Q Source: Chapter 10: India yearbook 2017

66 Consider the following about the programme ‘Padhe Bharat Badhe Bharat’.
1. It was launched as a nationwide sub-programme of Rashtriya Ucchatar Shiksha Abhiyaan.
2. It aims to improve enrolment ratio in secondary and higher education by way of active community intervention.
Which of the above is/are correct?
A. 1 only
B. 2 only
C. Both 1 and 2
D. None
User Answer :
Correct Answer : D
Answer Justification :

Justification: It is a sub-programme of Sarva Shiksha Abhiyaan named PBBB and was launched in 2014 to ensure
quality at the foundational years of schooling i.e., classes I and II.

(C) Insights Active Learning. | All rights reserved. www.insightsias.com 40


Total Marks : 200
TEST - 19 Mark Scored : 0

Through this programme it will be ensured that all children are able to read with comprehension as well as basic
numeracy skills. The programme envisages dedicated teachers for classes I and II.

It centres on capacity building of teachers, organizing separate reading periods in daily school time-table,
maintaining a print rich environment, for reading through children’s literature in school libraries and reading
corners in classes I and II; for tribal children special bridge materials have been prepared in states which have a
high tribal population.

Q Source: Chapter 10: India Yearbook 2017

67 Consider the following with reference to the University Grants Commission (UGC) and All India Council for
Technical Education (AICTE).
1. Both of them are statutory bodies.
2. Both of them are under the Ministry of Human Resource Development (MHRD).
3. Both of them have the authority to enforce any sanctions on colleges affiliated with the universities regulated by
them.
Select the correct answer using the codes below.
A. 1 and 2 only
B. 1 and 3 only
C. 2 only
D. 1, 2 and 3
User Answer :
Correct Answer : A
Answer Justification :

Justification: Statement 1: Established in November 1945 first as an advisory body and later on in 1987 given
statutory status by an Act of Parliament, AICTE is responsible for proper planning and coordinated development
of the technical education and management education system in India.

The AICTE accredits postgraduate and graduate programs under specific categories at Indian institutions as per its
charter.

UGC, a statutory body, provides recognition to universities in India, and disburses funds to such recognised
universities and colleges.

Statement 3: In a 2013 judgement the Supreme Court said "as per provisions of the AICTE Act and University
Grants Commission (UGC) Act, the council (AICTE) has no authority which empowers it to issue or enforce any
sanctions on colleges affiliated with the universities as its role is to provide guidance and recommendations.

Q Source: Chapter 10: India yearbook 2017

68 Consider the following with reference to the Economic Survey 2017-18.


1. It is published annually by the Department of Economic Affairs, Ministry of Finance.
2. It is presented only to the Lok Sabha before the presentation of the Annual Budget.
Which of the above is/are correct?
A. 1 only
B. 2 only
C. Both 1 and 2
D. None
User Answer :
Correct Answer : A

(C) Insights Active Learning. | All rights reserved. www.insightsias.com 41


Total Marks : 200
TEST - 19 Mark Scored : 0

Answer Justification :

Justification: Statement 1: It is the Finance ministry's view on the annual economic development of the country.

The Survey reviews the developments in the Indian economy in the last financial year, summarizes the
performance on major development programs, and highlights the policy initiatives of the government and the
prospects of the economy in the short to medium term.

Statement 2: This document is presented to both houses of Parliament during the Budget Session (before
presenting the budget).

Q Source: Improvisation: Chapter 1: Economic Survey 2017-18

69 The cave is carved into the hard monolithic granite rock face of Barabar hills, flanked to its left by the smaller
Sudama cave. The ornamentation on the "curved architrave" of the cave consists of carvings of elephants on their
way to the stupas. It refers to?
A. Kanheri Caves
B. Mogalarajapuram Caves
C. Udayagiri Caves
D. Lomus Rishi caves
User Answer :
Correct Answer : D
Answer Justification :

Learning: Lomas Rishi Cave is carved into the hard monolithic granite rock face of Barabar hills, flanked to its
left by the smaller Sudama cave.

This rock-cut cave was carved out as a sanctuary. It was built during the Ashokan period of the Maurya Empire in
the 3rd century BC, as part of the sacred architecture of the Ajivikas.

The hut-style facade at the entrance to the cave, which was meant as a milk shed, had an impact on South Asian
rock-cut architecture.

It became a model for all such arched entrance portals built at many other Buddhist and Jain caves in India, such
as the much bigger Buddhist Chaitya halls of Ajanta or Karli in Maharashtra.

Q Source: http://www.ccrtindia.gov.in/induscivilization.php

70 India’s economic openness is measured by


1. Volume of exports and imports
2. Private sector domestic investment growth
3. Ease of doing business index
Select the correct answer using the codes below.
A. 1 and 2 only
B. 2 and 3 only
C. 3 only
D. 1 only
User Answer :
Correct Answer : D
Answer Justification :

(C) Insights Active Learning. | All rights reserved. www.insightsias.com 42


Total Marks : 200
TEST - 19 Mark Scored : 0

Justification: The Openness Index is an economic metric calculated as the ratio of country's total trade, the sum of
exports plus imports, to the country's gross domestic product = (Exports Imports) / (Gross Domestic Product)

The interpretation of the Openness Index is: the higher the index the larger the influence of trade on domestic
activities, and the stronger that country's economy.

Ease of doing business index is a passive indicator of openness, and openness as such is not measured by it. So, 3
is wrong.

Q Source: Improvisation: Chapter 1: Economic Survey 2017-18

71 The Human Rights (Amendment) Act, 2006 brought which of these important changes in the functioning and/or
composition of the National Human Rights Commission?
1. It empowered the NHRC to transfer complaints received by it to the concerned SHRC.
2. It declared that Chairperson of the National Commission for the Scheduled Castes shall be deemed to be a member
of the NHRC.
Which of the above is/are correct?
A. 1 only
B. 2 only
C. Both 1 and 2
D. None
User Answer :
Correct Answer : C
Answer Justification :

Justification: It brought amendments in the Protection of Human Rights Act, 1993, relate to the following
important issues (only few listed here):

Changing the eligibility condition for appointment of member of SHRCs

Strengthening the investigative machinery available with Human Rights Commissions

Empowering the Commissions to recommend award of compensation, etc. even during the course of
enquiry

Empowering the NHRC to undertake visits to jails even without intimation to the state governments

Enabling the NHRC to transfer complaints received by it to the concerned SHRC

Providing that the Chairperson of the National Commission for the Scheduled Castes and the Chairperson
of the National Commission for the Scheduled Tribes shall be deemed to be members of the NHRC

Enabling the Central Government to notify future international covenants and conventions to which the Act
would be applicable

Q Source: Chapter on NHRC: Indian Polity: M Laxmikanth


(C) Insights Active Learning. | All rights reserved. www.insightsias.com 43
Total Marks : 200
TEST - 19 Mark Scored : 0

72 Consider the following with referene to Mughal school of painting.


1. It developed as a synthesis of the Indian style of painting and the Safavid school of Persian painting.
2. It is marked by naturalism portraying several natural themes even while being secular.
Which of the above is/are correct?
A. 1 only
B. 2 only
C. Both 1 and 2
D. None
User Answer :
Correct Answer : C
Answer Justification :

Justification: Statement 1: A large number of Indian artists from all over India were recruited to work under the
Persian masters appointed by Akbar for the development of paintings. The Mughal style thus evolved as a result of
a happy synthesis of the indigenous Indian style of painting and the Safavid school of Persian painting

Statement 2: The Mughal style is marked by supple naturalism based on close observation of nature and fine and
delicate drawing. It is of an high aesthetic merit. It is primarily aristocratic and secular.

Q Source: http://www.ccrtindia.gov.in/visualarts.php

73 By looking at sedimentary rocks of different ages, scientists can figure out how climate has changed through Earth’s
history. This is because
1. Sedimentary rocks contain fossils of organisms of previous ages.
2. Sedimentary rocks preserve a record of the environments that existed when they formed.
Which of the above is/are correct?
A. 1 only
B. 2 only
C. Both 1 and 2
D. None
User Answer :
Correct Answer : C
Answer Justification :

Justification: Statement 1: If certain organisms became extinct at a time, say due to severe drought or inter-glacial
shifts, this is recorded in sedimentary fossils as an event of a mass disappearance. Scientists can predict collapse of
certain species due to climate change this way.

Statement 2: Based on the above reasoning, sedimentary rocks will contain record of oceanic environments or
glacial environments or deserts.

This information can be decoded by studying various layers of sedimentary rocks and their arrangement in layers
deep down the crust.

Q Source: Chapter 2: Goh Cheng Leong - Certificate Physical and Human Geography

74 Consider the following with reference to the guidelines released by the Ministry of Parliamentary Affairs on Youth
Parliament.
1. The lay-out of the Chamber of their Youth Parliament should resemble as far as possible the lay-out of the
Chamber of Lok Sabha.
2. The first part of a sitting in the Youth parliament should be devoted to oral questions.
(C) Insights Active Learning. | All rights reserved. www.insightsias.com 44
Total Marks : 200
TEST - 19 Mark Scored : 0

3. In the Youth Parliament, the Parliamentary Business is arranged in the order in which it is shown in the
‘Direction by the Speaker’.
Select the correct answer using the codes below.
A. 1 and 2 only
B. 3 only
C. 1 and 3 only
D. 1, 2 and 3
User Answer :
Correct Answer : D
Answer Justification :

Justification: Statement 1: The working of Youth Parliament should broadly follow the pattern of the working of
Lok Sabha. Same goes for the physical layout and other things.

Statement 2: It should be devoted to oral question commonly known as Starred Questions in the Indian Parliament.

It is the most popular hour for the members of Parliament and also for the Public. From the point of view of a
Minister it is the most searching test of his grasp on the items or business allocated to him, his ability to lead as
well as his ready with when answering supplementary.

Statement 3: The order in which various items of business are normally taken up in the House is as follows:-

Oath of affirmation

Laying on the Table the President’s address to both Houses of Parliament

Obituary references and so on.

This resembles that of the Lok Sabha, which is decided by Lok Sabha Speaker and in the Youth parliament by the
concerned speaker.

Q Source: Improvisation: Chapter 3: India Yearbook 2016

75 Consider the following with reference to Swayam - Study Webs of Active - Learning for Young Aspiring Minds.
1. It is a Massive Open Online Courses (MOOCs) dedicated for secondary education in India.
2. It has been integrated as a core component of the Sarva Shiksha Abhiyaan (SSA).
Which of the above is/are correct?
A. 1 only
B. 2 only
C. Both 1 and 2
D. None
User Answer :
Correct Answer : D
Answer Justification :

Justification: Swayam is a MOOC initiative on a national platform with a comprehensive academic structure.

It is a MHRD initiative and offers courses for UG, PG and Diploma levels too. So, 1 is wrong.
(C) Insights Active Learning. | All rights reserved. www.insightsias.com 45
Total Marks : 200
TEST - 19 Mark Scored : 0

The integrated platform will offer courses covering engineering, humanities and social sciences etc., to be
used by learners at large.

Formation of a Consortium of premier educational institutions and universities to offer flipped online
courses instantaneously after due authentication and award of certification is one of the major planks of the
scheme.

Under SWAYAM, professors of centrally-funded institutions in India — such as Indian Institutes of


Technology (IITs), Indian Institutes of Management (IIMs), and central universities — will offer online
courses to citizens of India.

So, 2 is wrong, as it is not a part of SSA as such.

Q Source: Chapter 10: India yearbook 2017

76 Consider the following with reference to the Botanical Survey of India (BSI).
1. It was setup by the Wildlife Act with an important objective of identifying plant species with economic virtue.
2. It acts as the custodian of authentic collections in well planned herbaria and documenting plant resources.
Which of the above is/are correct?
A. 1 only
B. 2 only
C. Both 1 and 2
D. None
User Answer :
Correct Answer : B
Answer Justification :

Justification: Statement 1: The Botanical Survey of India (BSI) was established in 1890 with the objectives of
exploring the plant resources of the country and identifying plant species with economic virtue.

The Wildlife Act was enacted in 1972. So, 1 is wrong.

Statement 2: In 1954, the Government reorganised the BSI with the objectives of (1) undertaking intensive floristic
surveys and collecting accurate and detailed information on the occurrence, distribution, ecology and economic
utility of plants in the country; (2) collecting, identifying and distributing materials that may be of use to
educational and research institutions; and (3) acting as the custodian of authentic collections in well planned
herbaria and documenting plant resources in the form of local, district, state and national flora.

Q Source: Improvisation: Chapter 1 (Flora): India Yearbook 2016

77 Consider the following with reference to the External sector data of India since financial year 2008-09, when the
Global recession began.
1. Exports have consistently fallen since then.
2. Imports have consistently risen since then.
3. Trade deficit has continuously worsened since then.
Select the correct answer using the codes below.
A. 2 only
B. 1 and 3 only
(C) Insights Active Learning. | All rights reserved. www.insightsias.com 46
Total Marks : 200
TEST - 19 Mark Scored : 0

C. 1, 2 and 3
D. None of the above
User Answer :
Correct Answer : D
Answer Justification :

Justification: The table below shows the tabulation. You need not remember the data, but only the important and
broad trends.

Q Source: Chapter 1: Economic Survey 2017-18

78 India’s climate is largely affected by two seasonal winds - the north-east monsoon and the south-west monsoon.
Consider the following about them. Assertion (A): The north-east monsoon brings most of the rainfall in the country.
Reason (R): The north-east monsoon blows from sea to land, unlike the South-west Monsoon.
In the context of the above, which of these is correct?
A. A is correct, and R is an appropriate explanation of A.
B. A is correct, but R is not an appropriate explanation of A.
C. A is correct, but R is incorrect.
D. Both A and R are incorrect.
User Answer :
Correct Answer : D
Answer Justification :

Justification: The north-east monsoon is commonly known as the winter monsoon.

(C) Insights Active Learning. | All rights reserved. www.insightsias.com 47


Total Marks : 200
TEST - 19 Mark Scored : 0

It blows from land to sea whereas south-west monsoon known as summer monsoon blows from sea to land after
crossing the Indian Ocean, the Arabian sea and the Bay of Bengal.

The south-west monsoon brings most of the rainfall during the year in the country. SO, both A and R are wrong.

NE Monsoon brings light rainfall on the south-eastern coasts of the country, for e.g. in Tamil Nadu during the
winter season.

Q Source: Chapter 1: India Yearbook 2017

79 Consider these different styles of architecture as a part of Hindu temple architecture and the temples where they have
been used.
1. Gadag architecture A. Kasivisvesvara Temple
2. Maru-Gurjara architecture B. Konark Sun Temple
3. Kalinga architecture C. Chennakesava Temple
Select the correct match using the codes below.
A. 1-B, 2-C, 3-A
B. 1-A, 2-B, 3-C
C. 1-A, 2-C, 3-B
D. 1-C, 2-B, 3-A
User Answer :
Correct Answer : C
Answer Justification :

Justification: Use elimination. Sun temple Konark is a famous example of Kalinga architecture, located in
Odisha. There is only one option where 3-B is mentioned, which is C.

Statement 1: The Gadag style of architecture is also called Western Chalukya architecture. The style flourished for
150 years (1050 to 1200 CE). Some examples are the Saraswati temple in the Trikuteshwara temple complex at
Gadag, the Doddabasappa Temple at Dambal, the Kasivisvesvara Temple at Lakkundi, and the Amriteshwara
temple at Annigeri.

Statement 2: It originated somewhere in the 6th century in and around areas of Rajasthan. It shows the deep
understanding of structures and refined skills of Rajasthani craftmen of bygone era.

Statement 3: The design which flourished in eastern Indian state of Odisha and Northern Andhra Pradesh are
called Kalinga style of architecture.

The prominent examples of Rekha Deula are Lingaraj Temple of Bhubaneswar and Jagannath Temple of Puri. One
of the prominent example of Khakhara Deula is Vaital Deula.

Q Source: Improvisation: http://ccrtindia.gov.in/templearchitecture.php

80 The ‘National Agricultural Science Fund’ (NASF) was setup with which of the following objective(s) and aim(s)?
1. To provide policy support to the decision makers for use of basic and strategic research in agriculture
2. To venture fund local innovations and green solutions proposed by self-governance bodies like Gram Sabha and
Gram Panchayat
Which of the above is/are correct?
A. 1 only
B. 2 only
C. Both 1 and 2

(C) Insights Active Learning. | All rights reserved. www.insightsias.com 48


Total Marks : 200
TEST - 19 Mark Scored : 0

D. None
User Answer :
Correct Answer : A
Answer Justification :

Justification: Statement 1: The main objective of the scheme has been to build capacity for basic, strategic and
cutting edge application research in agriculture and address issues which can be solved by intensive basic and
strategic research jointly by team of organizations/ institutions.

Statement 2: It essentially aims to foster research and a research culture that will use and advance the frontiers of
scientific knowledge to effectively meet the present, anticipated and unanticipated problems of agriculture through
various modes and critical investments in research projects.

It also seeks to build a storehouse of advancement of knowledge in science related to agriculture and awareness of
the national importance of basic and strategic research in agriculture.

Q Source: Chapter 4: India Yearbook 2016

81 Which of the following features were typical features of Islamic architecture in India?
1. Use of arches and vaults in construction
2. Worship of human figures on sculptures
3. Utilisation of kiosks and tall towers
Select the correct answer using the codes below.
A. 2 only
B. 1 and 2 only
C. 1 and 3 only
D. 1, 2 and 3
User Answer :
Correct Answer : C
Answer Justification :

Justification: Statement 1 and 3: The Islamic style also incorporated many elements from the traditional Indian
style and a compound style emanated. The introduction of decorative brackets, balconies, pendentive decorations,
etc in the architecture is an example in this regard. The other distinguishing features of Indo-Islamic architecture
are the utilisation of kiosks (chhatris), tall towers (minars) and half-domed double portals.

Statement 2: Human worship and its representation are not allowed in Islam, so the buildings and other edifices are
generally decorated richly in geometrical and arabesque designs.

Learning: Even though lime was known and to certain extent used in construction work in India fairly early, mud
was generally used for brick work and large blocks of stones were laid one on top of the other and held by means
of iron clamps.

The Muslims, like the Romans, were also responsible for making extensive use of concrete and lime mortar as an
important factor of construction and incidentally used lime as plaster and a base for decoration which was incised
into it and held enamel work on tiles.

Q Source: http://ccrtindia.gov.in/indoislamicarchitecture.php

82 Among the following, which one has the largest river basin?
(C) Insights Active Learning. | All rights reserved. www.insightsias.com 49
Total Marks : 200
TEST - 19 Mark Scored : 0

A. Indus Basin
B. Ganga Basin
C. Brahmaputra Basin
D. Krishna-Godavari Basin combined
User Answer :
Correct Answer : B
Answer Justification :

Learning: River basin is considered as the basic hydrological unit for planning and development of water
resources.

There are 12 major river basins with catchment area of 20000 sq. km and above.

The major river basin is the Ganga-Brahmaputra-Meghna , which is the largest with catchment area - more than
43% of the catchment area of all the major rivers in the country.

The other major river basins with catchment area more than 1.0 lakh sq. km are Indus, Mahanadi, Godavari and
Krishna.

All major river basins and many medium river basins are inter-state in nature which cover about 81% of the
geographical area of the country.

Q Source: Improvisation: Chapter 1: India Yearbook 2017

83 Credit rating of India, published by organizations such as S&P and Moody’s essentially measures?
A. GDP growth numbers
B. Credibility of sovereign bonds
C. Fiscal deficit sustainability in terms of borrowing
D. Stability of money supply in the economy
User Answer :
Correct Answer : B
Answer Justification :

Learning: Credit rating is an estimate of the ability of a person or organization to fulfil their financial
commitments, based on previous dealings.

To determine a bond's rating, a credit rating agency analyzes the accounts of the issuer and the legal agreements
attached to the bond to produce what is effectively a forecast of the bond's chance of default.

The metrics vary somewhat between the agencies. S&P's ratings reflect default probability, while ratings by
Moody's reflect expected investor losses in the case of default.

Standard & Poor's credit rating for India stands at BBB- with stable outlook.

Q Source:
http://www.livemint.com/Politics/eAg9P9FVGY22gcpV4paYcI/SPs-credit-rating-for-India-unchanged-outlook-st
able.html

84 ‘Mahapurushalakshana’, in the context of religious history of India, is associated with


A. Symbols related to Gautam Buddha
B. Invocation ceremony at the Gomateshwara statue

(C) Insights Active Learning. | All rights reserved. www.insightsias.com 50


Total Marks : 200
TEST - 19 Mark Scored : 0

C. Ashwamedha rituals mentioned in Rig Veda


D. Astrological predictions given by Maharishi Parashara
User Answer :
Correct Answer : A
Answer Justification :

Learning: The first century of the Christian era's revolutionary change, had far-reaching effects, not only on the
art of India, but also on the artistic development of Buddhist countries of Asia.

Buddha who was hitherto designated only by a symbol, was conceived in human form. His person was
given some of the 32 suspicious bodily signs associated with the Mahapurushalakshana, such as the
protuberance of the skull, the hair-knot, bindi between the eyebrows and elongated ears.

This change came about as a result of the new changes that had crept into the religious outlook of Buddhism
due to the influence of the Devotional School of Hindu Philosophy, requiring the worship of personal gods.

Possibly, the emergence of the image of Buddha in Gandhara and in Mathura was a parallel development.
In each case, it was produced by the local artist craftsmen working in the local tradition.

Q Source: http://ccrtindia.gov.in/buddhistsculp.php

85 It is the traditional buffalo race and bullock-cart racing of coastal regions of Karnataka. It was recently banned by the
Karnataka High Court in view of SC’s ban on Jallikattu. The state assembly, however, has recently passed a bill to
allow the sports. It is?
A. Atya Patya
B. Chaturanga
C. Kambala
D. Langdi
User Answer :
Correct Answer : C
Answer Justification :

Learning: Kambala Kambala is an annual traditional Buffalo Race (he-buffalo) held in coastal districts of
Karnataka to entertain rural people of the area. Slushy/marshy paddy field track is used for Kambala.

The sports season generally starts in November and lasts till March. The contest generally takes place
between two pairs of buffaloes, each pair race in two separate wet rice fields tracks, controlled by a whip-
lashing farmer.

There is also ritualistic approach as some agriculturists race their he-buffaloes for thanks giving to god for
protecting their animals from diseases.

But in recent times, Kambala has become an organised rural sport.

In 2016, the Karnataka high court had banned the kambala along with bull-cart race in the state while

(C) Insights Active Learning. | All rights reserved. www.insightsias.com 51


Total Marks : 200
TEST - 19 Mark Scored : 0

hearing a case filed by the People for the Ethical Treatment of Animals (PETA) citing animal cruelty.

Karnataka Assembly has passed Prevention of Cruelty to Animals (Karnataka Amendment) Bill, 2017 to
allow kambala, the traditional buffalo race and bullock-cart racing.

The bill seeks to exempt kambala and bullock-cart racing from the ambit of Prevention of Cruelty to
Animals (PCA) Act, 1960.

Governor may refer the Bill to President for his assent as it amends central law.

Q Source:
http://www.thehindu.com/news/national/karnataka/Karnataka-amends-law-to-allow-kambala/article17296265.ece

86 Consider the following about the National Authority, Chemical Weapons Convention (NACWC).
1. It was set up by a resolution of Cabinet Secretariat.
2. It is chaired by the Prime Minister of India.
3. It is housed under the Ministry of Chemicals and Fertilizers.
Select the correct answer using the codes below.
A. 1 and 2 only
B. 1 only
C. 2 only
D. 2 and 3 only
User Answer :
Correct Answer : B
Answer Justification :

Justification: It was set up in 1997 to fulfil the obligations enunciated in the Chemical Weapons Convention.

It prohibits the development, production, execution, transfer, use and stockpiling of all chemical weapons
by Member-States in a non-discriminatory manner.

The Parliament enacted in 2000 a CWC Act which came into force in 2015 to give effect to the provisions
of the Chemical Weapons Convention in India.

In accordance with the provisions of this Act a high-level steering committee under the Chairmanship of the
Cabinet Secretary with Secretary (Chemical and Petrochemicals), Foreign Secretary, Secretary, Defence
Research and Development, Defence Secretary and Chairman, National Authority as its other members,
oversees the functioning of the National Authority.

The National Authority is responsible for implementation of the CWC Act, liaison with Organisation for the
Prohibition of Chemical Weapons (OPCW) and other state parties, fulfilling of declaration obligation,
negotiating facility agreements etc.

Q Source: Chapter 3: India Yearbook

(C) Insights Active Learning. | All rights reserved. www.insightsias.com 52


Total Marks : 200
TEST - 19 Mark Scored : 0

87 The Electronics Development Fund (EDF), setup by the Union Government, works with venture capitalists to create
funds, known as ‘daughter funds,’ which
1. Provide risk capital to companies developing new technologies
2. Aim at achieving the larger goal of “Net Zero Imports― by 2020
Which of the above is/are correct?
A. 1 only
B. 2 only
C. Both 1 and 2
D. None
User Answer :
Correct Answer : C
Answer Justification :

Justification: As part of the "Digital India" agenda of the Government, and to develop the Electronics System
Design and Manufacturing (ESDM) sector so as to achieve the net Zero Imports goal.

Setting up of Electronic Development Fund (EDF) is one of the important strategies adopted by DeitY
which would enable creating a vibrant ecosystem of innovation, research and development (R&D) and with
active industry involvement.

It is with this objective that an Electronic Development Fund (EDF) is set up as a “Fund of Funds” to
participate in professionally managed “Daughter Funds” which in turn will provide risk capital to
companies developing new technologies in the area of electronics, nano-electronics and Information
Technology (IT).

The EDF will also help attract venture funds, angel funds and seed funds towards R&D and innovation in
the specified areas. It will help create a battery of Daughter funds and Fund Managers who will be seeking
good start-ups (potential winners) and selecting them based on professional considerations.

Q Source:
http://indianexpress.com/article/business/budget/union-budget-2017-personal-finance-railways-digital-india-demo
netisation-social-sector-all-you-need-to-know/

88 At present 106 districts in 10 States have been identified by the Government of India as Left Wing Extremism
(LWE) affected districts in the country. Which of the following are NOT among those districts?
A. Bastar, Guntur, Ramgarh
B. Mirzapur, Bankura, Jehanabad
C. Warangal, Hazaribagh, Nalanda
D. Jamshedpur, Chittor, Kanchipuram
User Answer :
Correct Answer : D
Answer Justification :

Justification: Such questions can be solved by elimination.

Kanchipuram is in Tamil Nadu and TN is not a listed state among those 10 states. This clearly points to option D.

The 10 states are Andhra Pradesh, Telangana, Chattisgarh, Bihar, Jharkhand, West Bengal, Odisha, MP, UP and

(C) Insights Active Learning. | All rights reserved. www.insightsias.com 53


Total Marks : 200
TEST - 19 Mark Scored : 0

Maharashtra.

You can find the entire list of districts here http://pib.nic.in/newsite/PrintRelease.aspx?relid=136706

Q Source: Improvisation: Chapter 9: India Yearbook 2017

89 Weathering of rocks can be caused by


1. Seepage of water
2. Thermal stresses
3. Plant root penetration in rocks
4. Frost action
Select the correct answer using the codes below.
A. 2 and 3 only
B. 1 and 4 only
C. 2, 3 and 4 only
D. 1, 2, 3 and 4
User Answer :
Correct Answer : D
Answer Justification :

Justification: Statement 1 and 4: Water seeps into cracks and crevices in rock. If the temperature drops low
enough, the water will freeze. When water freezes, it expands.

The ice then works as a wedge. It slowly widens the cracks and splits the rock. When ice melts, water performs the
act of erosion by carrying away the tiny rock fragments lost in the split.

Statement 2: Thermal stress weathering results from the expansion and contraction of rock, caused by temperature
changes.

For example, heating of rocks by sunlight or fires can cause expansion of their constituent minerals.

As some minerals expand more than others, temperature changes set up differential stresses that eventually cause
the rock to crack apart.

Statement 3: Plants and animals are agents of mechanical weathering. The seed of a tree may sprout in soil that has
collected in a cracked rock.

As the roots grow, they widen the cracks, eventually breaking the rock into pieces. Over time, trees can break apart
even large rocks. Even small plants, such as mosses, can enlarge tiny cracks as they grow.

Q Source: Chapter 4: Goh Cheng Leong - Certificate Physical and Human Geography

90 Consider the following about Prime Minister’s Award for Excellence in Public Administration.
1. These awards are given on Independence Day each year.
2. Awards can be given only to individual officers working at either the Central, State or District levels.
3. Recommendations for the award to the Prime Minister are made by a Joint Committee of Parliament.
Select the correct answer using the codes below.
A. 1 and 3 only
B. 2 only
C. 2 and 3 only
D. None of the above

(C) Insights Active Learning. | All rights reserved. www.insightsias.com 54


Total Marks : 200
TEST - 19 Mark Scored : 0

User Answer :
Correct Answer : D
Answer Justification :

Justification: Statement 1: PM’s Awards are given on Civil Services Day held every 21st April of the year.

Statement 2: On the occasion of Civil Services Day 2016 awards were conferred to the 10 best performing districts
for excellence done in implementation of four priority programmes namely Pradhan Mantri Jan Dhan Yojna,
Swachh Bharat Gramin, Swacith Vidyalaya Abhiyan and Soil Health care Scheme.

So, it can be given to organizations as well as districts. It has been given to individuals earlier too.

Statement 3: The application for the award is evaluated by three hierarchical committees. The Screening
Committee chaired by the Additional Secretary level officer,

Expert Committee chaired by Secretary, Department of Administrative Reforms and Public Grievances and
the Empowered Committee chaired by the Cabinet Secretary. The empowered panel then makes
recommendations to the PM for the award.

The award carries with it i) a Medal ii) a Scroll, iii) an incentive of 10 lakh to the awarded
district/organisation to be utilized for implementation of project/programme or bridging resource gaps in
any area of activity of public welfare.

Q Source: Chapter 3: India Yearbook 2016

91 ‘Superbugs’ sometimes seen in news are related to


A. Cyber Hacking protocols
B. Multi-drug resistant bacteria
C. Malicious open source codes
D. Special evolutionary species of bovines
User Answer :
Correct Answer : B
Answer Justification :

Learning: Resistance arises through one of three ways: natural resistance in certain types of bacteria, genetic
mutation, or by one species acquiring resistance from another.

Resistance can appear spontaneously because of random mutations; or more commonly following gradual
buildup over time, and because of misuse of antibiotics or antimicrobials.

Resistant microbes are increasingly difficult to treat, requiring alternative medications or higher doses, both
of which may be more expensive or more toxic.

Microbes resistant to multiple antimicrobials are called multidrug resistant (MDR); or sometimes superbugs

According to the World Health Organization (WHO), infections caused multi-drug resistant bacteria kills

(C) Insights Active Learning. | All rights reserved. www.insightsias.com 55


Total Marks : 200
TEST - 19 Mark Scored : 0

around 700,000 people each year.

This figure is predicted to rise 10 million by 2050 justifying warnings of so-called antibiotic apocalypse
which could be among the 21st Century’s greatest threats.

Q Source: http://www.bbc.com/news/world-australia-38977259

92 Ocean acts as a large carbon sink on earth due to


A. Its large geographical coverage
B. Rich population of phytoplankton and seagrass
C. Difference in the partial pressure of carbon dioxide between seawater and air
D. All of the above
User Answer :
Correct Answer : D
Answer Justification :

Justification & Learning: Concepts in Option A and B have been explained before.

Option C: Carbon dioxide readily dissolves in water and the oceans provide a huge reservoir of carbon. Across the
world's oceans there is a continual cycle of equilibration of dissolved carbon dioxide in water with carbon dioxide
in the atmosphere.

The difference in partial pressure of the CO2 between seawater and air facilitate gaseous exchange. This allows
atmospheric CO2 to dissolve in seawater.

The carbon dioxide which dissolves in our oceans occurs in three main forms. Aside from the normal carbon
dioxide form, it is also found as bicarbonate and carbonate ions.

Most exists as bicarbonate with carbonate ions acting as the link between carbon dioxide and bicarbonate.

As concentrations of carbon dioxide increase the supply of carbonate ions becomes limited and so the oceans
become less and less able to take up carbon dioxide from the atmosphere.

Q Source:
https://www.theguardian.com/environment/climate-consensus-97-per-cent/2017/feb/16/scientists-study-ocean-abs
orption-of-human-carbon-pollution

93 What is/are the key difference(s) between Production Sharing Contract (PSC) model and Revenue Sharing Model of
oil block investments?
1. In the former, entire investment is made by the government while operations are handled by the private sector.
2. In the latter, government assures a minimum level of profits or returns to the investors.
Which of the above is/are correct?
A. 1 only
B. 2 only
C. Both 1 and 2
D. None
User Answer :
Correct Answer : D
Answer Justification :

(C) Insights Active Learning. | All rights reserved. www.insightsias.com 56


Total Marks : 200
TEST - 19 Mark Scored : 0

Justification: Developing the oil and gas sector needs huge investment. Therefore, the government invites private
companies under PPP to invest in oil and gas blocks alongwith providing technology; and reap benefits.

There are two models that are followed worldwide -

Production-sharing model

Revenue Sharing Model

Statement 1: As per the PSC model, firms will first develop oil and gas fields. They will then first recover their
investment and operating expenditure. Then they will share profits with the government. So, 1 is wrong as private
sector invests too.

Statement 2: In the Revenue Sharing model, firms will develop the oil and gas fields. But the revenue will be
shared between the government and firms from the first batch of production of the oil and gas itself (irrespective of
cost recovery or not). It will be on the basis of level of output in the block, not on the level of investment. So, 2 is
wrong as profits are not ensured, however there is no mandatory payment on the part of developers.

Q Source: http://pib.nic.in/newsite/PrintRelease.aspx?relid=158489

94 Consider the following statements about mountain ranges of India.


1. The Vindhyas are an unbroken continuous chain of mountain anticlinal ridges.
2. The Aravalli range are the oldest fold mountains in India rising upwards at a faster than Himalayas due to tectonic
instability.
Which of the above is/are correct?
A. 1 only
B. 2 only
C. Both 1 and 2
D. None
User Answer :
Correct Answer : D
Answer Justification :

Justification: Statement 1: The Vindhyas do not form a single range in the proper geological sense: the hills
collectively known as the Vindhyas do not lie along an anticlinal or synclinal ridge.

The Vindhya range is actually a group of discontinuous chain of mountain ridges, hill ranges, highlands and
plateau escarpments. The term "Vindhyas" is defined by convention, and therefore, the exact definition of the
Vindhya range has varied at different times in history.

Statement 2: old Fold Mountains are characterized by having stopped growing higher due to the cessation of
upward thrust caused by the stopping of movement of the tectonic plates in the Earth's crust below them.

In ancient times they were extremely high but since have worn down almost completely by millions of years of
weathering. In contrast, the Himalayas are continuously rising young fold mountains of today.

Q Source: Improvisation: Chapter 1: India Yearbook 2017

95 Consider the following about the Employees’ Provident Fund Organisation (EPFO).

(C) Insights Active Learning. | All rights reserved. www.insightsias.com 57


Total Marks : 200
TEST - 19 Mark Scored : 0

1. It was established by statutory law.


2. The fund is administered by a private fund management firm hired on an annual bid basis by the government.
3. It is under the administrative control of Ministry of Labour and Employment.
Select the correct answer using the codes below.
A. 1 and 3 only
B. 2 and 3 only
C. 2 only
D. 1 only
User Answer :
Correct Answer : A
Answer Justification :

Justification: Statement 1: The Employees’ Provident Fund came into existence with the promulgation of the
Employees Provident Funds Ordinance in 1951. It was replaced by the Employees’ Provident Funds Act, 1952.

Statement 2: The Act and Schemes framed there under are administered by a tri-partite Board known as the
Central Board of Trustees, Employees’ Provident Fund, consisting of representatives of Government (Both Central
and State), Employers, and Employees.

The Board administers a contributory provident fund, pension scheme and an insurance scheme for the workforce
engaged in the organized sector in India. It is one of the world’s largest organizations in terms of clientele and the
volume of financial transactions undertaken by it.

Statement 3: The Board is assisted by the Employees’ PF Organization (EPFO). The EPFO is under the
administrative control of Ministry of Labour and Employment, Government of India (click here).

EPFO is also the nodal agency for implementing Bilateral Social Security Agreements with other countries on a
reciprocal basis. The schemes cover Indian workers as well as International workers (for countries with which
bilateral agreements have been signed.

Q Source: Improvisation: Chapter 1: Economic Survey 2017-18

96 A comet tail and coma is formed largely due to


1. Vapourisation of volatile materials within the comet by solar radiation
2. Accumulation of ice on the top layer of comets
Which of the above is/are correct?
A. 1 only
B. 2 only
C. Both 1 and 2
D. None
User Answer :
Correct Answer : A
Answer Justification :

Justification: Statement 1: A comet tail—and coma—are features visible in comets when they are illuminated by
the Sun and may become visible from Earth when a comet passes through the inner Solar System.

As a comet approaches the inner Solar System, solar radiation causes the volatile materials within the comet to
vaporize and stream out of the nucleus, carrying dust away with them. Separate tails are formed of dust and gases,
becoming visible through different phenomena; the dust reflects sunlight directly and the gases glow from
ionisation. So, 1 is correct.

(C) Insights Active Learning. | All rights reserved. www.insightsias.com 58


Total Marks : 200
TEST - 19 Mark Scored : 0

Statement 2: In the outer Solar System, comets remain frozen and are extremely difficult or impossible to detect
from Earth due to their small size.

As they get closer to the Sun, ice starts to melt and the glow increases. So, 2 is wrong.

Q Source: Improvisation: Chapter 1: Goh Cheng Leong - Certificate Physical and Human Geography

97 India has joined ‘Network for Improving Quality of Care for Maternal, Newborn and Child Health’, a global health
network focused on improving the quality of care for new mothers and babies. Consider the following about it.
1. The network is supported by WHO and United Nations International Children’s Fund (UNICEF).
2. The network membership legally commits India to match global healthcare service norms to improve health
security in India.
Which of the above is/are correct?
A. 1 only
B. 2 only
C. Both 1 and 2
D. None
User Answer :
Correct Answer : A
Answer Justification :

Justification: There are other partners, apart from WHO and UNICEF. However, there is no legal commitment, so
statement 2 is wrong.

Learning: India is among nine countries that will be part of the network. Other countries are Bangladesh, A Cote
d’Ivoire, Ethiopia, Ghana, Malawi, Nigeria, Tanzania and Uganda.

Under this network, countries will work to improve the quality of health care facilities to mothers and
babies.

They will strengthen national efforts to end preventable deaths of pregnant women and newborns by 2030.

(C) Insights Active Learning. | All rights reserved. www.insightsias.com 59


Total Marks : 200
TEST - 19 Mark Scored : 0

Under it, counties will build and strengthen their national institutions, identify quality of care focal points at
all levels of the health system.

They will also accelerate and sustain the implementation of quality-of-care improvement packages for
mothers, newborns and children.

They will also strengthen capacity and motivation of health professional to plan and manage quality
improvement, improve data collection and increase access to medicines, equipment, clean water and
supplies.

Q Source:
http://indianexpress.com/article/india/india-joins-un-network-to-halve-maternal-newborn-deaths-4525667/

98 Consider the following statements about the Cabinet Secretariat.


1. It functions directly under the Ministry of Personnel and Public Grievances.
2. It is responsible for the administration of the Government of India (Transaction of Business) Rules.
3. It ensures that the President, the Vice-President and Ministers are kept informed of the major activities of all
ministries and departments.
4. The administrative head of the Secretariat is the ex-officio Chairman of the Civil Services Board.
Select the correct answer using the codes below.
A. 1 and 4 only
B. 2, 3 and 4 only
C. 2 and 3 only
D. 1, 2, 3 and 4
User Answer :
Correct Answer : B
Answer Justification :

Justification: Statement 1: The Cabinet Secretariat functions directly under the Prime Minister.

Statement 2 and 3: The business allocated to Cabinet Secretariat is (i) Secretarial assistance to Cabinet and Cabinet
Committees; and (ii) Rules of Business.

The Cabinet Secretariat is responsible for the administration of the Government of India (Transaction of
Business) Rules, 1961 and the Government of India (Allocation of Business) Rules, 1961, facilitating
smooth transaction of business in ministries/departments of the Government by ensuring adherence to these
rules.

It assists in decision-making by ensuring inter-ministerial coordination, ironing out differences amongst


ministries/departments and evolving consensus through the instrumentality of the standing and ad hoc
Committees of Secretaries.

Management of major crisis situation in the country and coordinating activities of various ministries in such
a situation is also one of the functions of the Cabinet Secretariat.

Statement 4: The administrative head of the Secretariat is the Cabinet Secretary who is also the Chairman of the

(C) Insights Active Learning. | All rights reserved. www.insightsias.com 60


Total Marks : 200
TEST - 19 Mark Scored : 0

Civil Services Board.

Q Source: Improvisation: Chapter 3: India Yearbook 2016

99 'Tithi Bhojan', is a community participatory model linked to which of these flagship schemes?
A. National Food Security Act (NFSA)
B. Integrated Child Development Services (ICDS)
C. Mid-Day Meal Scheme (MDM)
D. National Food Security Mission (NFSM)
User Answer :
Correct Answer : C
Answer Justification :

Learning: It was initially followed by the Gujarat government in the midday meal programme, which was later
replicated across the country by MHRD (in 2014).

Under this initiative, food is voluntarily served among school children by villagers in several forms like sweet and
namkeen with the regular midday meal and supplementary nutritive items like sprouted beans.

The objective is to inculcate the feeling of equity and brotherhood among the children of all communities and
reduction of the gap between the school administration and the community.

All the states/UTs have been requested to consider to adopt the practice of Tithi Bhojan with this nomenclature or
any nomenclature suitable to the state governments. The concept has been adopted by different states with local
nomenclatures like ‘Sampriti Bhojan’ in Assam.

Q Source: Chapter 10: India Yearbook 2017

100 Consider the following about Soil-Transmitted Helminths (STH).


1. STH worms interfere with nutrient uptake in small children causing parasitic infestation.
2. No medical solution for alleviation or treatment of this disease exists as of now.
3. India has the highest burden of STH in the world.
Select the correct answer using the codes below.
A. 1 and 3 only
B. 2 and 3 only
C. 3 only
D. 1 and 2 only
User Answer :
Correct Answer : A
Answer Justification :

Justification: Statement 1: It is caused by different species of roundworms. They are among the most common
worms infections worldwide caused specifically by those worms which are transmitted through soil contaminated
with faecal matter.

These worms interfere with nutrient uptake in small children causing parasitic infestation. They live in human
intestines and consume nutrients meant for the human body.

Statement 2: India celebrates National Deworming day each year. On this day, Albendazole tablets are given to all
targeted children.

(C) Insights Active Learning. | All rights reserved. www.insightsias.com 61


Total Marks : 200
TEST - 19 Mark Scored : 0

The goal of the day is to deworm all preschool and school-age children between the ages of 1-19 years in
order to improve their overall health, cognitive development, nutritional status and quality of life.

This year for the first time, deworming tablets were given to the private schools children also.

The parasitic infestation or diseases lead to severe complications among the children resulting in anemia,
malnutrition and improper mental and physical development.

Statement 3: According to WHO, India has the highest burden of STH in the world, with estimated 220 million
children (aged 1-14) to be at risk of worm infections.

Q Source: National Deworming Day

(C) Insights Active Learning. | All rights reserved. www.insightsias.com 62

You might also like